You are on page 1of 74

Trauma and Burns MCQ

1. Nasotracheal intubation:

A. Is preferred for the unconscious patient without cervical spine injury.

B. Is preferred for patients with suspected cervical spine injury.

C. Maximizes neck manipulation.

D. Is contraindicated in the patient who is breathing spontaneously.

Answer: B

DISCUSSION: The first principle in the management of any injured patient is to secure an adequate
airway. This can be particularly difficult in the presence of facial or laryngeal trauma, or in the
unconscious patient with a suspected cervical spine injury. The mechanical removal of oral debris
followed by the “chin lift” or “jaw thrust” maneuvers to relieve soft tissue obstruction of the pharynx are
the first steps. However, when there is any question regarding the adequacy of the airway, or in the
presence of severe head injury, or when the patient is in profound shock, more definitive airway control
is required. In most patients this involves oral endotracheal intubation. However, the insertion of an oral
endotracheal tube often involves hyperextension of the neck with the potential for aggravating cervical
spine ligamentous or bony injury. Nasotracheal intubation is the preferred option for the patient with
suspected cervical spine ligamentous or bony injury since the head and neck can be maintained in the
neutral position with minimal manipulation. This technique requires a breathing patient, as the passage
of air must be heard through the nasotracheal tube prior to its insertion through the larynx into the
trachea. Nasotracheal intubation is contraindicated in the presence of mid-face fractures. In this
situation, a surgical airway (cricothyroidotomy, tracheostomy, or needle cricothyroidotomy) is the
preferred option.

2. Cardiac contusions caused by blunt chest trauma:

A. Are fairly easy to diagnose.

B. Occur in up to 20% to 40% of patients with major blunt thoracic trauma.

C. Do not usually cause right ventricular dysfunction.

D. Demonstrate arrhythmia as the most common complication.

Answer: BD
DISCUSSION: Cardiac contusions are often difficult to diagnose, but have been estimated to occur in 5%
of major trauma patients, and up to 20% to 40% of patients with severe blunt chest injury. The difficulty
in diagnosing cardiac contusions is that they remain a pathologic diagnosis, confirmed only at autopsy or
on direct cardiac examination. The injury may vary from superficial epicardial petechiae to complete
transmural damage. Although significant myocardial injuries, such as ventricular rupture, coronary
vessel thrombosis, and valvular disruption, have been reported, the most common clinically significant
result of cardiac contusion is the occurrence of arrhythmias. Hence, an initial electrocardiogram (ECG)
and subsequent continuous cardiac monitoring for at least 24 hours is generally recommended.
Alternative methods of diagnosing myocardial contusion include creatine phosphokinase cardiac
isoenzymes (CPK-MB), two-dimensional echocardiography, gated ventricular scintigraphic angiography
(GVA), radioactive thallous chloride ( 201Tl) uptake, and right ventricular monitoring. Unfortunately,
none of these tests is adequately sensitive or specific in the diagnosis of cardiac contusion, and their
correlation with the presence of arrhythmias or ECG changes is also imprecise.

3. According to the recommendations of the American College of Surgeons Committee on Trauma,


which of the following patients should be transported to a trauma center?

A. Fifty-year-old female who fell 8 feet from a step ladder, with isolated hip fracture and normal vital
signs.

B. Fifteen-year-old bicyclist with closed head injury and Glasgow Coma Scale score of 12.

C. Twenty-three-year-old male assault victim with stab wound to the back, normal vital signs, and
respiratory distress.

D. Three-year-old infant passenger (restrained) in motor vehicle accident with normal vital signs and no
apparent injuries except abdominal wall contusion.

Answer: BCD

DISCUSSION: The American College of Surgeons Committee on Trauma has developed a field triage
decision scheme to help identify trauma victims with a significant risk of dying as a result of their
injuries. This classification is based on four factors: (1) abnormal physiologic signs, (2) anatomic area of
injury, (3) mechanism of injury, and (4) concurrent or co-morbid disease states. Major physiologic
abnormalities include a Glasgow Coma Scale score of less than 13, systolic blood pressure less than 90
mm. Hg, respiratory rate less than 10 or greater than 29 per minute, or a Revised Trauma Score of less
than 11 or a Pediatric Trauma Score of less than 9. Significant anatomic considerations include
penetrating injuries to the torso, head and neck, and proximal extremities, flail chest, combination of
trauma with burns to greater than 10% of body surface area, two or more proximal long bone fractures,
pelvic fractures, paralysis, or traumatic amputation above the wrist or ankle. Significant mechanisms of
injury include a death in the same passenger compartment or ejection from the automobile, high-impact
(greater than 5 miles per hour) auto-pedestrian injuries, or a pedestrian thrown or run over. The co-
morbid factors include pediatric or elderly (<5 or >55) patients or known history of insulin-dependent
diabetes or cardiac, respiratory, or psychotic disorders. These criteria should serve as guidelines for
medical control and the pre-hospital care providers. Such triage guidelines have been shown to produce
the triage of only a small fraction (5% to 10%) of all injured patients to Level I or Level II trauma centers.

4. Which of the following statements about head injuries is/are false?

A. The majority of deaths from auto accidents are due to head injuries.

B. Head injury alone often produces shock.

C. A rapid and complete neurologic examination is part of the initial evaluation of the trauma patient.

D. Optimizing arterial oxygenation is part of initial therapy.

Answer: B

DISCUSSION: Head injuries cause the majority of deaths following automobile accidents, with rupture of
the thoracic aorta the second most common cause of fatality. Head injury itself rarely produces
hypotensive shock. It is only in the terminal phases of brain death that hypotension may be attributable
to head injury alone. Therefore, hypotension in trauma patients must be assumed to be secondary to
volume depletion or ongoing hemorrhage. An occult site of hemorrhage (chest, abdomen, pelvis,
retroperitoneum, or extremities) must be strongly suspected and dealt with accordingly. A rapid and
complete neurologic assessment is a crucial part of the initial assessment of all trauma patients. This
initial exam gives an excellent indication of injury severity and prognosis. Since the ultimate outcome of
a brain injury is dependent on adequate cerebral perfusion and oxygenation, adequate airway control,
ventilation, hemorrhage control, volume restitution, and arterial oxygenation are crucial factors in the
early management of head injuries.

5. Which of the following statements about maxillofacial trauma is/are false?

A. Asphyxia due to upper airway obstruction is the major cause of death from facial injuries.
B. The mandible is the most common site of facial fracture.

C. The Le Fort II fracture includes a horizontal fracture of the maxilla along with nasal bone fracture.

D. Loss of upward gaze may indicate either an orbital floor or orbital roof fracture.

Answer: B

DISCUSSION: Maxillofacial injuries generally do not cause life-threatening injuries, with the exception of
those that occlude the airway. Therefore, the first priority in assessing and managing the patient with
maxillofacial trauma is to assess and assure the adequacy of the airway. The face is typically divided into
thirds when defining injuries. Injuries to the upper third of the face are often accompanied by ocular or
central nervous system complications as well as facial deformities. Fractures of the orbital roof are
frequently associated with frontal sinus and nasal ethmoid fractures, and are accompanied by a loss of
upward gaze due to involvement of the superior rectus muscle. However, the most common cause of
loss of upward gaze is orbital floor injury and associated entrapment of the globe or injury to the inferior
rectus muscle. Middle third of facial structures include the maxilla, zygoma, orbits, and nose. The Le Fort
classifications of facial fractures are commonly employed to describe these complex fracture lines. In a
Le Fort II fracture, the superior fracture line is transverse through the nasal bones or through the
articulation of the maxillary and nasal bones with the frontal bones. This is also known as the
“pyramidal” fracture of the mid-face. The diagnosis is established by digital manipulation of the anterior
maxilla and observation for mobility of the central triangle (the maxilla and nose). The lower third of the
face contains a single facial bone, the mandible. After the nasal bones, the mandible is the second most
commonly fractured facial bone.

6. What percentage of patients with thoracic trauma require thoracotomy?

A. 10%–15%.

B. 20%–25%.

C. 30%–40%.

D. 45%–50%.

Answer: A

DISCUSSION: Twenty-five per cent of civilian trauma deaths are caused by thoracic trauma, and two
thirds of these deaths occur after the patient reaches the hospital. Mortality of hospitalized patients
with isolated chest injury ranges from 4% to 8% and increases to 35% when multiple additional organ
systems are involved. Despite high mortality, only 10% to 15% of thoracic injuries require thoracotomy.
Most injuries are successfully managed by the rather simple life-saving maneuvers of airway control and
tube thoracotomy. Unrelenting hemorrhage following either penetrating or blunt thoracic trauma is a
primary indication for immediate thoracotomy. An initial thoracic blood loss of greater than 1500 ml.
(30% of blood volume) or an ongoing loss of 250 ml. for 3 consecutive hours serves only as a practical
guideline. The patient's hemodynamic status and overall condition should be the most influential
factors.

7. The radiographic findings indicating a torn thoracic aorta include:

A. Widened mediastinum.

B. Presence of an apical “pleural cap.”

C. First rib fractures.

D. Tracheal deviation to the right.

E. Left hemothorax.

Answer: ABCDE

DISCUSSION: All of the listed radiographic findings should arouse suspicion of a possible torn thoracic
aorta. The most common abnormality noted is a widening of the mediastinal shadow, although only 20%
to 40% of patients with a wide mediastinum have aortic injury. In addition to the radiographic signs
listed, other findings that may alert the physician to the possibility of an aortic tear include loss of aortic
contour, elevation of the left mainstem bronchus, depression of the right mainstem bronchus, shift of
the nasogastric tube to the left, and the presence of retrocardiac density. Aortography remains the
“gold standard” diagnostic modality and is indicated if aortic injury is suspected on the basis of
mechanism of injury and any of these suggested findings.

8. Which of the following statements about diagnostic peritoneal lavage (DPL) is/are false?

A. DPL is the diagnostic procedure of choice for gunshot wounds to the abdomen with no obvious intra-
abdominal injuries.
B. The average reported incidence of false-positive DPL in patients with significant pelvic fractures is
20% to 30%.

C. Accuracy rates for DPL have generally been reported between 95% and 97%.

D. DPL has been entirely replaced by computed tomography as the diagnostic procedure of choice
following blunt abdominal trauma.

Answer: AD

DISCUSSION: DPL remains the most sensitive and specific indicator of intra-abdominal injury in the
trauma patient. The accuracy rates for DPL in several large collective series reveal an overall sensitivity
of 95%, specificity of 98% to 99%, and overall accuracy of 97%. As result, DPL remains the mainstay for
diagnosis of intraperitoneal injury in the trauma patient; however, not every trauma patient requires
DPL. In the awake, alert, and responsive patient with isolated abdominal injuries, the physical
examination and history are very helpful in predicting the presence of significant injury. In the patient
with lower torso (nipples to pubis) or back or flank gunshot wounds, the incidence of intra-abdominal
injury is so high that exploratory laparotomy without further diagnostic modalities is generally
advocated. In addition, DPL is generally inaccurate in the diagnosis of retroperitoneal injuries
(duodenum, renal, pancreas), and significant retroperitoneal hemorrhage in association with pelvic
fractures produces a false-positive DPL rate of up to 30%. Computed tomography (CT) scans have
proved extremely valuable in these situations. General recommendations for the use of abdominal CT
scans in trauma victims include patients who are hemodynamically stable (normal) with (1) equivocal
abdominal examination, (2) closed head injury, (3) spinal cord injury, (4) hematuria, and (5) pelvic
fractures with significant bleeding. These five indications are appropriate if the patient is truly
hemodynamically stable and the time required to perform CT does not delay any surgical procedures.

9. A 28-year-old male was injured in a motorcycle accident in which he was not wearing a helmet. On
admission to the emergency room he was in severe respiratory distress and hypotensive (blood pressure
80/40 mm. Hg), and appeared cyanotic. He was bleeding profusely from the nose and had an obviously
open femur fracture with exposed bone. Breath sounds were decreased on the right side of the chest.
The initial management priority should be:

A. Control of hemorrhage with anterior and posterior nasal packing.

B. Tube thoracostomy in the right hemithorax.

C. Endotracheal intubation with in-line cervical traction.

D. Obtain intravenous access and begin emergency type O blood transfusions.


E. Obtain cross-table cervical spine film and chest film.

Answer: C

DISCUSSION: Airway remains the first priority in the management of any patient with multiple injuries.
Control of the airway in a patient with head, face, and neck injury can be extremely challenging. In the
patient presented, the best option given for control of the airway is endotracheal intubation with in-line
cervical traction. This requires at least two persons, one to maintain the head in the neutral position and
one to insert the endotracheal tube under direct vision. An alternative in this case would be emergency
cricothyroidotomy, tracheostomy, or needle-jet insufflation. Nasotracheal intubation is not an option in
the presence of a mid-face fracture and a nasal hemorrhage. Clearly, attention must also be directed at
assuring adequacy of ventilation (potential right pneumothorax), assessing and treating obvious
hemorrhage, determining if there is occult intra-abdominal or thoracic hemorrhage, and determining
the patient's neurologic status. While management of these other issues can occur simultaneously, they
do not take priority over securing an adequate airway. In this patient the airway is so tenuous that time
should not be spent obtaining a cross-table cervical spine film and chest film prior to definitive control of
the airway.

10. True or False?

A. Trauma is second only to congenital heart disease as the leading cause of death in children.

B. Each year in the United States, approximately 50,000 people die from injuries.

C. Motor vehicle accidents (MVAs) involving intoxicated drivers are responsible for 50% of all MVA
fatalities.

D. Active prevention strategies (e.g., seat belts, helmets) have not proved effective in reducing injuries
and fatalities.

E. Falls and diving accidents comprise approximately 30% to 40% of cervical spine injuries.

Answer: TRUE C; FALSE ABD

DISCUSSION: The statistics on injuries highlight trauma as “the principal public health problem in
America today.” Trauma remains the leading cause of death in children and adults up to the age of 44
years, and injuries kill more Americans age 1 to 34 years than all diseases combined. Each year more
than 140,000 Americans die of injuries, 50,000 due to motor vehicle accidents. Just over 50% of motor
vehicle injuries involve intoxicated drivers. Injury prevention would be the most cost-effective method
of dealing with this major social and economic burden. Active injury prevention strategies are those that
require active continued cooperation on the part of the individual, such as wearing a helmet when
driving a motorcycle or wearing seat belts in automobiles. Passive approaches such as fitting all motor
vehicles with driver air bags require little or no individual cooperation and have clearly proved the more
effective option, but active prevention strategies have repeatedly been demonstrated to reduce injury
fatalities. States with seat belt and child restraining laws show an increase in seat belt use of more than
60% with a concurrent 9% to 12% reduction in occupant fatalities. Motor vehicle accidents are
responsible for approximately 60% of spinal cord injuries, falls for 20% to 30%, and diving accidents for
an addition 5% to 10%. Spinal cord injury acute care and rehabilitation represent some of the most
expensive medical treatment, with an average hospital charge of $50,000 in 1988 for a quadriplegic
survivor.

11. Regarding the diagnosis and treatment of cardiac tamponade, which of the following statements
is/are true?

A. Accumulation of greater than 250 ml. of blood in the pericardial sac is necessary to impair cardiac
output.

B. Beck's classic triad of signs of cardiac tamponade include distended neck veins, pulsus paradoxicus,
and hypotension.

C. Approximately 15% of needle pericardiocenteses give a false-negative result.

D. Cardiopulmonary bypass is required to repair most penetrating cardiac injuries.

Answer: C

DISCUSSION: Cardiac tamponade is most frequently caused by penetrating thoracic injury, but may
occasionally be observed following blunt thoracic trauma from cardiac chamber rupture, coronary artery
laceration, or ascending dissection of an aortic tear. Accumulation of as little as 150 ml. of blood in the
pericardium will sufficiently decrease diastolic filling to produce distended neck veins, cyanosis, and
decreased cardiac output. Beck's classic triad of distended neck veins, muffled heart sounds, and
hypotension is present in only one third of patients with tamponade. Pulsus paradoxicus is even less
frequently discernible. Immediate temporary treatment consists of pericardiocentesis, which also
provides a diagnosis. However, approximately 15% of pericardiocenteses give false-negative results
because of a clotted hemopericardium. Therefore, echocardiography prior to needle aspiration is
generally advisable if promptly available. In the patient in extremis, emergency thoracotomy with
pericardiotomy and cardiac repair should be performed. Most patients with penetrating cardiac wounds
do not require cardiopulmonary bypass to repair their injuries.
12. Which of the following statements or descriptions typically characterizes the syndrome of
overwhelming postsplenectomy sepsis?

A. A syndrome of fulminant gram-negative bacteremia and septicemia in asplenic individuals,


characterized by the presence of as many as 10 6 bacterial organisms per cu. mm. circulating in the
bloodstream.

B. A syndrome caused primarily by impaired host ability to mount an effective humoral


(immunoglobulin) response to infection.

C. A syndrome that occurs in 5% to 7% of patients following traumatic splenectomy.

D. A syndrome of rapidly appearing septic shock unresponsive to antibiotic therapy, with an average
mortality of 50%.

E. The syndrome may be prevented by preserving as little as 15% of splenic mass in adult trauma
victims.

Answer: D

DISCUSSION: In 1952 King and Schumaker suggested that children who had undergone splenectomy
were at risk for the development of bacterial infections, and the syndrome of overwhelming
postsplenectomy sepsis (OPSS) was suggested by Diamond in 1969. The syndrome is unlike fulminating
bacteremias and septicemia in individuals with normal splenic function. The onset is sudden, with
nausea, vomiting, headache, and confusion leading to coma. The new infecting organism is a gram-
positive organism in over half the cases, primarily Streptoccoccus pneumoniae. Blood cultures may
occasionally demonstrate up to as many as 10 6 bacterial organisms per cu. mm. circulating in the
bloodstream. Disseminated intravascular coagulation is common along with hypoglycemia, electrolyte
imbalance, and shock unresponsive to antibiotics and fluid or pharmacologic support. Mortality has
generally been reported as high as 50% and even up to 80% for pneumococcal infections. The true
incidence of overwhelming postsplenectomy sepsis following a splenectomy from trauma is not well
defined. Green and colleagues suggested that the risk of OPSS is 166 times the rate expected for the
general population. Eraklis and Filler suggested that the incident rate of mortality from sepsis and OPSS
is 78 times greater than that expected for the general population. Despite this increased frequency,
overwhelming postsplenectomy sepsis remains a rare event. Singer's large review of 688 children who
had undergone splenectomy for trauma demonstrated only a 1.45% incidence of postsplenectomy
sepsis, but a 40% mortality. The occurrence of OPSS appears to be less following splenectomy for
trauma when compared with splenectomy for congenital hematologic disorders. Nonetheless, the
recognition of the severe nature of this entity has prompted many trauma surgeons to more
aggressively attempt splenic salvage. Animal laboratory evidence suggests that at least 50% of the
splenic tissue mass must be preserved to prevent overwhelming postsplenectomy sepsis. The
immunologic function of the spleen that appears to be most beneficial in preventing OPSS is the spleen's
capacity for clearance of blood-borne particles and the provision of circulating opsins, which assist in
cell-mediated immunologic functions.

13. Trauma deaths most commonly occur at three distinct time periods after injury. Which of the
following statement(s) is/are true concerning the time pattern of trauma mortality?

a. Only 10% of trauma deaths occur within seconds or minutes of the injury

b. A second mortality peak occurs within hours of injury with deaths in this time period being markedly
reduced with the development of trauma and rapid transport systems

c. Death one day to weeks after the injury are almost entirely due to infection and multiple organ failure

d. Late mortality in trauma patients, occurring days to weeks after the injury, has not been affected by
better trauma delivery systems

Answer: b

Trauma deaths occur at three traditionally recognized times after injury. About half of all trauma-related
deaths occur within seconds or minutes of injury and are related to lacerations of the aorta, heart, brain
stem, brain, and spinal cord. Few of these patients are saved by health care systems, regardless of
efficiency. The second mortality peak occurs within hours of injury and accounts for about 30% of
deaths, half of which are due to hemorrhage and half due to central nervous system injuries. Important
reductions in mortality during this period have resulted from the development of trauma and rapid
transport systems. Overall, trauma mortality rates have been reduced from about 30% to 2% to 9%
where well-organized trauma care systems exist. The third mortality peak includes deaths that occur
one day after trauma to weeks later. This mortality rate is usually attributed to infection and multiple
organ failure. Ten to 20% of trauma deaths occur during this period. The development of efficient
trauma systems, however, has changed the epidemiology of these deaths. During the first week after
trauma, refractory intracranial hypertension after severe head injury now accounts for a significant
number of these deaths. The incidence of sepsis and multiple organ failure has diminished as the result
of aggressive and better early resuscitation and care. Sepsis and multiple organ failure now account for
about 5% of overall mortality and only 30% of late mortality where organized trauma systems exist.
14. Which of the following statement(s) is/are true concerning the epidemiology of trauma?

a. Trauma is the leading cause of death of individuals less than 44 years of age

b. Trauma follows only cancer and heart disease as leading causes of productive life lost

c. Motor vehicle accidents are the most common cause of traumatic death in young males of all ethnic
groups

d. Young males are the population at highest risk for trauma death

Answer: a, d

Although injury affects all age groups, it is epidemic within the younger population of our society. In the
United States, injury is the leading cause of death in individuals less than 44 years and results in 70% of
the total hospital admissions. Young males are the highest risk group, not because of physiologic
distinctions, but because of their propensity to engage in high-risk activities. Although the three leading
causes of traumatic death in all ethnic groups are motor vehicle accidents, homicide, and suicide, for
individuals under 35 years of age, the order in which these occur differs. In the African-American
population, the leading cause of death in this age group is homicide, while in all other groups it is motor
vehicle accidents. Although morbidity and mortality figures are important, another important method of
analyzing the toll injury places on a society is in years of productive life lost. Years of productive life lost
is used to reflect the amount of productive working time lost due to premature death. Since injury is so
prevalent in the younger population, a traumatic death in this age group will result in a large number of
years of productive life lost, more so than deaths in the older age groups due to chronic diseases. In fact,
years of productive life lost due to injury are approximately 40% higher than those found in cancer or
heart disease patients, the second and third leading causes of productive life lost.

15. Which of the following statement(s) is/are true concerning the biomechanics of blunt trauma?

a. A small child and a large adult have a markedly different level of energy transfer in a high speed
vehicular collision

b. Shear strain injuries result from rapid acceleration or deceleration

c. Tensile strain results from direct compression of tissues


d. The tolerance of biologic tissue to trauma injury is directly proportional to the elasticity of the organ

Answer: b, c, d

The severity of any injury is directly proportionate to the amount of kinetic energy transferred to the
tissues and the properties of that tissue which accept and dissipate the energy. Kinetic energy (KE) is a
function of the mass (M) of an object and its

velocity (V):

KE = M x V^2 /2

It is clear from this relationship that changes in velocity alter the kinetic energy transferred more
significantly than changes in mass. Therefore, a small child and a large adult, though significantly
different in size and weight, are subjected to similar levels of energy transfer in a high-speed vehicular
collision, the primary determinant being velocity rather than mass. The tolerance of a biologic tissue to
traumatic injury is directly proportional to the elasticity of the organ—that is, its ability to return to its
original shape and position. Elasticity is directly affected by the rate of loading, or the rate at which
strain is applied to the tissues. Applying the force more rapidly increases the likelihood of exceeding
tolerance. Blunt trauma results in two types of forces during impact. First, changes in speed
(acceleration or deceleration) create shear strain, and second, deformity changes (stretch or
compression) creates tensile strain.

16. The patient described above has also suffered major facial trauma. Which of the following
statement(s) is/are true?

a. A frontal bone fracture and injury to the frontal sinus is a common facial injury in a young adult

b. The optic nerve can be injured in a LeFort type II fracture

c. A facial nerve injury may occur with the fracture of the temporal bone

d. Coronal CT scan images can be a useful adjunct to the evaluation of the patient with facial and head
injuries

Answer: c, d
A major cause of maxillofacial trauma are motor vehicle accidents. Facial skeletal fractures and soft
tissue damage in the frontal, orbital, nasal, zygomatic, maxillary and mandibular regions are included.
The frontal bone, which houses the frontal sinuses, is particularly strong due to its arched configuration
as well as thick, hard bone. The amount of force necessary to fracture the frontal sinus is two to three
times greater than that necessary for other facial bone fractures. Consistent fracture patterns from
blows to the maxilla have been classified by LeFort and occur within and along the maxilla at its junction
with weaker and aerated bone of the paranasal sinuses and nasal cavity. The classic LeFort fractures are
classified as LeFort I, LeFort II and LeFort III and are of increasing complexity and morbidity. The
cribriform plate, ethmoidal arteries, optic nerve and internal maxillary artery are all vulnerable to injury
with a LeFort III fracture.

Soft tissue injuries of the face are encountered even more often than facial fractures. The facial nerve is
the most important underlying structure at risk since blunt or penetrating trauma to the nerve or
branches can cause complete or partial ipsilateral facial paralysis. The most common cause of facial
nerve injury is fracture of the temporal bone, but injury can occur anywhere from the intracranial to the
extracranial facial course of the nerve.

After securing the airway and controlling life-threatening hemorrhage, the secondary survey including
the facial area is carried out. The nose is inspected for deformity, pain, mobility, septal hematoma and
obstruction. Bleeding should be managed immediately. Leakage of cerebral spinal fluid suggests a
cribriform plate or ethmoidal fracture and a presence should warn against insertion of any nasal tubes
or packing. Since CT scan is part of the standard management of the head-injured patient, sections of
the facial skeleton can be obtained simultaneously, providing information on the extent of facial
fractures in addition to the status of the brain. Axial and coronal sections (obtained with the patient’s
head hanging with the neck extended) are complimentary and are especially helpful in delineating the
cribriform plate and ethmoid roof region, the orbital rims, and the overall vertical facial height.

17. There are a number of options for resuscitative fluids. Which of the following statement(s) is/are
true concerning fluids used for resuscitation of shock?

a. Resuscitation with crystalloid requires volume replacement in a ratio of 1:1 to volume lost

b. The literature strongly supports the use of colloid as being superior to crystalloid in the resuscitation
of shock

c. Risks of autotransfused blood include disseminated intravascular coagulation and activation of


fibrinolysis
d. Hypertonic saline solution results in volume expansion, an increase in left ventricular performance,
decreased peripheral resistance, and redistribution of cardiac output to kidneys and viscera

e. The use of perfluorocarbons as an experimental resuscitative fluid has been demonstrated to


stimulate the immune system

Answer: c, d

Balanced salt solutions are the most commonly used resuscitative fluids, and their use to restore
extracellular volume significantly decreases the transfusion requirement after hemorrhagic shock.
Lactated Ringers and normal saline are the most effective crystalloid solutions in common use.
Resuscitation with crystalloid require a volume administration ratio of 3:1 to 4:1 over volume lost.
Although colloids do not replete the interstitial space, they have a volume-expanding effect somewhat
greater than the amount used. Colloids commonly used for volume expansion in hypovolemia include
albumen, dextran 70, dextran 40, and hydroxyethyl starch (hetastarch). Significant controversy exists
concerning the use of crystalloid versus colloid resuscitation. Although the question has not been
resolved, several recent studies have indicated an advantage to crystalloid in resuscitation. A meta-
analysis of colloid versus crystalloid resuscitation after hemorrhagic shock has demonstrated a higher
mortality rate in the colloid resuscitated patients, partly due to pulmonary complications. Patients who
lose more than 25 to 30% of total blood volume will need blood for resuscitation. Type O, Rh-negative
(universal donor blood) is immediately available without a cross match. Type-specific blood is available
within most blood banks within five to ten minutes of receipt of the blood specimen, while the patient is
being resuscitated with balanced salt solutions. Although not cross matched, this blood can be
administered safely, and therefore its rapid availability and safety make type-specific blood the blood of
choice for resuscitation in trauma. Autotransfusion involves the collection of shed blood and its
reinfusion through a filter back into the patient. Autotransfused blood may produce disseminated
intravascular coagulation (DIC) and activation of fibrinolysis. In addition, blood collected from the
peritoneal cavity after hollow viscus injury, even with cell washing, may lead to bacterial contamination
of the autotransfused blood. Hypertonic solutions have been used in the resuscitation of patients after
burn, shock, elective vascular surgery and trauma. In addition to volume expansion, hypertonic saline
solutions have been shown to increase left ventricular performance, decrease peripheral resistance
from arteriolar dilatation, and redistribute cardiac output to the kidneys and viscera. Perfluorocarbons
are an experimental resuscitation fluid comprised of large, branched or cyclic aliphatic compounds
which have the ability to dissolve and carry oxygen. Although effective in volume resuscitation with
improved oxygen delivery and oxygen-carrying capacity, perfluorocarbon infusion has been shown to
depress platelet counts, plasma immune globulin levels and depress other aspects of immune function.
18. Hemorrhage initiates a series of compensatory responses. Which of the following statement(s) is/are
true concerning the physiologic responses to hemorrhagic shock?

a. An immediate response is an increased sympathetic discharge with resultant reflex tachycardia and
vasoconstriction

b. Transcapillary refill is a response serving to restore circulating volume

c. Extracellular fluid becomes increasingly hyperosmolar

d. Adrenergically mediated vasoconstriction is well maintained at the arteriolar and precapillary


sphincters

Answer: a, b, c

Hemorrhage initiates both rapid and slower, more sustained compensatory responses. The body
responds to maintain hemostasis almost immediately after the onset of hemorrhage. Decreased
activation of the arterial baroreceptors, though a decrease in blood pressure or even more subtly, a
decrease in pulse pressure, causes an increased sympathetic discharge, resulting in reflex tachycardia
and vasoconstriction. Increased adrenergic output with increased secretion of catecholamines also leads
to vasoconstriction, increased heart rate, and increased myocardial contractility. Sustained
compensatory responses include the release of vasoactive hormones and fluid shifts from the
interstitium and the intracellular space. Adrenergically mediated vasoconstriction affects arterial
precapillary and postcapillary sphincters and small veins and venules. The decrease in intravascular
hydrostatic pressure distal to the precapillary sphincter leads to reabsorption of interstitial fluid into the
vascular space and thereby functions to restore circulating volume. This is known as transcapillary refill.
The increased release of stress hormones coupled with relative insulin resistance after shock leads to
high extracellular glucose concentrations. In addition, products of anaerobic metabolism from
hypoperfused cells accumulate in the extracellular compartment, inducing hyperosmolarity. This
extracellular hyperosmolarity draws water from the intracellular space, increasing interstitial osmotic
pressure, which in turn drives water, sodium and chloride across the capillary endothelium into the
vascular space. If the shock state continues, however, the postcapillary sphincter remains in spasm, but
the arteriolar and precapillary sphincters cannot maintain the tension, and they become relaxed. As
sphincters relax, the capillary hydrostatic pressure increases and sodium, chloride and water move into
the interstitium leading to further depletion of intravascular volume.

19. Which of the following steps is/are part of the primary survey in a trauma patient?
a. Insuring adequate ventilatory support

b. Measurement of blood pressure and pulse

c. Neurologic evaluation with the Glasgow Coma Scale

d. Examination of the cervical spine

Answer: a, b, c

The resuscitation team’s first priority is to simultaneously assess the airway, blood pressure and level of
consciousness of the patient. The first priority is assessment of the airway. After establishment of an
airway, the next priority is to insure adequate ventilatory exchange by rapid auscultation of both lung
fields and assessment for mechanical factors that may interfere with breathing. After establishment of
an airway, ventilation and appropriate pleural drainage, if necessary, the next priority is the assessment
of the patient’s circulatory status. This includes an estimation of blood volume and cardiac function. The
initial survey evaluates blood pressure, pulse, and skin perfusion. It is important to emphasize that
effective resuscitation from hemorrhagic shock requires both restoration of intravascular volume and
control of hemorrhage. The final priority of the primary survey is a brief neurological evaluation using
the components of the Glasgow Coma Scale. Although maintaining axial immobilization of the cervical
spine is an important early component of all assessments and resuscitation protocols, examination of
the cervical spine regardless of injury is part of the secondary survey.

20. Immediate life-threatening injuries that preclude air exchange which can be treated in the field
include which of the following?

a. Tension pneumothorax

b. Massive open chest wounds

c. Sucking chest wounds

d. Tracheal disruption

Answer: a, b, c
After establishment of a patent and controlled airway, the next priority is to insure that air exchange is
taking place. Immediate life-threatening injuries that preclude air exchange include: tension
pneumothorax, massive open chest wounds, sucking chest wounds, and tracheal disruption. There are
no maneuvers likely to correct tracheal disruption in the field. Both open chest wounds and sucking
chest wounds will respond to endotracheal intubation and positive pressure ventilation. Tension
pneumothorax may require field decompression in the rare patient. Field techniques to deal with
tension pneumothorax include needle thoracostomy and chest tube thoracostomy.

21. Which of the following statement(s) is/are true concerning the diagnosis of a peripheral vascular
injury?

a. The presence of a Doppler signal over an artery in an extremity essentially rules out an arterial injury

b. Doppler examination is a valuable tool in the diagnosis of venous injuries

c. A gunshot wound in the proximity of a major vessel is an absolute indication for arteriography

d. Both the sensitivity and specificity of arteriography of the injured extremity approaches 100%

Answer: d

Segmental arterial pressure determination by Doppler technique is a valuable adjunct to the physical
examination of extremity vascular trauma. The presence of audible Doppler signals over an artery in the
extremity does not rule out an arterial injury or indicate adequate perfusion. In the healthy and
normovolemic person, the normal ankle-brachial index is 1:1. A ratio less than 0.9 or a 20-mm Hg
difference between extremities should arouse the suspicion of significant arterial trauma. Doppler
examination has not been widely used to screen for significant venous injuries and is of unproven value.
The selective use of arteriography is fundamental to the evaluation of patients with suspected vascular
trauma. The indications for arteriography generally have included a history of moderate hemorrhage at
the penetrating injury site, injury in proximity to major arterial structures, diminished pulses, and
peripheral nerve injury in the distribution of a nerve that is in proximity to a major vessel. Proximity as
the sole indication for arteriography in the absence of diminished ankle-brachial ratio or other signs of
major trauma, has proven to be an unreliable indicator of the need for arteriography. In the absence of
classical signs of major vascular injury, patients with penetrating wounds in proximity to major vessels
may be observed closely without arteriography. The use of arteriography can significantly reduce the
rate of unnecessary exploration for suspected vascular trauma. If routine surgical exploration is
performed whenever vascular injury is suspected, a negative exploration rate of about 60% or more can
be expected. Selective use of arteriography reduces the negative exploration rate to about 35%.
Arteriography is an extremely reliable method of excluding vascular trauma. In this context, the
sensitivity is 97% to 100% and the specificity is 90% to 98%, with an overall accuracy between 92% and
98%.

22. A 22-year-old male is hospitalized with multiple extremity fractures including a comminuted fracture
of the femur and multiple rib fractures. Which of the following statement(s) is/are true concerning his
hospital course?

a. Low-dose heparin should not be employed during his hospital stay

b. Acute respiratory failure associated with petechiae of the head, torso, and sclerae would suggest a
pulmonary embolism

c. Early fracture fixation would decrease the incidence of fat emboli

d. The placement of a Greenfield filter should be avoided due to the risk of lower extremity edema

Answer: c

Perhaps the most catastrophic post-injury complication is pulmonary embolism. All patients with
orthopedic injury, particularly those with fractures of the lower extremities or pelvis, are at high risk for
deep venous thrombosis and subsequent pulmonary embolism. Prophylaxis with sequential
compression devices or low dose heparin has reduced the incidence of deep venous thrombosis in this
group. Although concern for the use of heparin is appropriate, prospective studies demonstrate that
low-dose heparin therapy can begin safely within 24 hours in 37% of patients and within 48 hours in
75%. Trauma patients who are paralyzed or immobilized by head injury, spine injury, or multiple
orthopedic injuries should be considered for placement of a Greenfield filter.

Fat embolism syndrome is a classic triad of acute respiratory failure; altered mental status, and
petechiae of the head, torso and sclerae; and is frequently associated with long-bone and pelvic
fractures. Less fulminant presentations, without petechiae and with lesser degrees of pulmonary
dysfunction, are more common. At present, the only therapy for fat emboli syndrome is supportive care.
Therefore, prevention is critical and numerous studies indicate that early fracture fixation decreases the
incidence of this and other pulmonary complications. However, a subset of patients with femoral
fractures and coexisting lung contusion has been recently found to have a higher incidence of ARDS if
the fracture is repaired early than if repaired late.
23. A middle-aged construction worker had a significant fall on the job and presents with obvious high
cervical spine injury. Which of the following statement(s) is/are true concerning his diagnosis and
management?

a. A paradoxical breathing pattern in which the abdomen protrudes on inhalation may be observed

b. If the patient appears well compensated on initial evaluation, intubation is unlikely to be necessary

c. The presence of hypotension strongly suggests significant blood loss from associated injury

d. The patient’s extremities are likely to appear warm and well perfused despite the presence of
hypotension

e. The use of methylprednisolone beginning 24 hours after the injury will be indicated

Answer: a, d

Fractures to the axial spine, especially in the high cervical spine, can cause varying degrees of respiratory
compromise. Patients with ventilatory failure from acute cord injury typically present with a paradoxical
breathing pattern in which the abdomen protrudes on inhalation, creating a see-saw appearance. This is
caused by paralysis of abdominal musculature and is seen with injuries as low as T-10 to T-11. Early
endotracheal intubation and mechanical ventilation must be considered, even in patients who appear
compensated on initial evaluation. There is a strong tendency for such patients to tire and develop
respiratory failure a few hours after the injury.

In addition to ventilatory compromise, high axial spinal lesions can cause significant hypotension,
confusing the initial evaluation of the patient. Most CNS control of arterial tone is mediated through the
sympathetic nervous system. In high thoracic and cervical spinal cord injuries, these controlling
pathways may be interrupted, with subsequent loss of vasomotor tone. This results in hypotension even
without significant blood loss. Unlike hypovolemic shock, the patient’s extremities are warm and well
perfused.

A prospective, randomized trial has suggested that high doses of methylprednisolone given within 8
hours of injury have improved neurologic recovery. Starting treatment with steroids more than 8 hours
after injury results in worse recovery than the placebo and is not recommended.
24. Which of the following statement(s) is/are true concerning Emergency Room thoracotomy?

a. Overall survival rates approach 25%

b. Blunt trauma patients without signs of life upon arrival in the Emergency Room are candidates for
Emergency Room thoracotomy

c. All patients with penetrating trauma to the chest and the absence of vital signs are candidates for ER
thoracotomy

d. None of the above

Answer: d

A recent meta-analysis of 24 reports concerning the outcome of Emergency Room thoracotomy found
that the overall survival rate was 11%. There were no survivors among patients with no signs of life
(supraventricular electrical activity, pupillary reaction, and agonal respirations) at the scene. In addition,
there were no neurologically intact survivors among blunt trauma patients without signs of life upon
arrival in the Emergency Department. Considering these findings, an appropriate algorithm would
indicate that Emergency Room thoracotomy for penetrating trauma is indicated only if patients had
signs of life at the scene and had lost signs of life less than five minutes prior to arrival in the Emergency
Room. Blunt trauma patients would be allowed Emergency Room thoracotomy only if the patient had
signs of life upon arrival at the Emergency Room. If patients meet these criteria and lose cardiac
function, airway placement and fluid resuscitation is initiated simultaneously with or immediately
followed by left anterior thoracotomy, pericardiotomy, and internal cardiac massage.

25. An untreated or an unrecognized compartment syndrome produces nerve and muscle damage and
prevents good functional recovery despite the patency of vascular repair. Which of the following factors
suggests the need for a fasciotomy?

a. A period of 6 hours or more between injury and restoration of perfusion

b. Combined arterial and venous injuries

c. Postoperative signs of muscle pain or pain on passive stretch

d. Elevated compartment pressures


answer: a, b, c, d

Factors that suggest the need for fasciotomy are as follows:

1. Prolonged period (6 hours or more) between injury and restoration of perfusion

2. Associated crush injury

3. Preoperative calf swelling

4. Combined arterial and venous injuries

5. Extensive venous ligation

6. Postoperative signs or disproportionate muscle pain, pain on passive stretch, or tender and firm
muscles

7. Elevated compartment pressures

26. Which of the following statement(s) is/are true concerning the consequences of vascular injuries?

a. Outcome is time-dependent

b. Further injury can take place after restoration of blood flow

c. Acute acidosis, hyperkalemia and myoglobin-induced renal failure can be consequences of severe
extremity ischemia

d. Ischemia to peripheral nerves and muscles can be tolerated to up to four hours without permanent
injury

Answer: a, b, c, d

Local consequences of vascular injuries are generally related to acute arterial occlusion from thrombosis
after injury. The results of ischemia distal to the injury sites may lead to limb or organ loss. The degree
of tissue loss is related to the adequacy of collateral flow, the sensitivity of distal tissue to ischemia, and
the delay involved in repairing the injury and restoring blood flow. With regard to these latter issues, the
variability is great. The brain is more sensitive to ischemia because of high basal energy requirements in
the absence of glycogen stores. Brain ischemia for longer than 4 minutes results in irreversible injury.
The nerves and muscles are much more resilient, tolerating periods of ischemia up to 4 hours without
permanent injury. An important principle of vascular repairs, however, is that the outcome is time-
dependent, necessitating an aggressive approach and a high priority.

The mechanism of injury from acute arterial ischemia includes both the initial anoxic phase when blood
flow is ceased and reperfusion phase after restoration of blood flow. Termed the reperfusion injury, this
phase includes the production of toxic metabolites and an inflammatory response which causes
significant endothelial damage. The events associated with restoration of arterial blood flow after
complete ischemia extend the magnitude and severity of the original insult in skeletal muscle and
peripheral nerves. If the severity of ischemia is significant enough to cause skeletal muscle necrosis,
rhabdomyalysis with the release of potassium and myoglobin into the systemic circulation follows.
Acute acidosis, hyperkalemia, and myoglobin-induced renal failure can occur.

27. Which of the following statement(s) is/are true concerning the surgical management of vascular
injuries?

a. A direct approach through the site of injury is often effective as the initial step

b. Systemic heparinization must be avoided in patients with multiple injuries

c. Reversed saphenous vein from the same extremity is the first choice as an interposition graft for
extensive arterial injuries

d. Venous repair should not be attempted in a hemodynamically unstable patient

Answer: b, d

The goal of operative management of vascular injuries is the rapid control of hemorrhage and the
restoration of perfusion, with salvage of extremity or organ in jeopardy. In isolated-extremity vascular
injury with arterial occlusion, systemic heparin should be administered to avoid propagation of
thrombus in vessels distal to the occlusion. In multiple-injury patients, especially those with central
nervous system trauma, heparin is inappropriate. The initial steps in the surgical management of
vascular injuries is to obtain proximal and distal control of the injured vessel. This is most easily
accomplished through uninjured areas adjacent to the injury using incisions normally employed for
elective exposure of these vessels. Direct approach to the site of injury is fraught with the hazards of
severe hemorrhage and iatrogenic trauma to the vessel itself or adjacent nerves. The management of
the arterial injury is determined by the extent of injury. In a repair of more extensive arterial injuries,
reversed saphenous vein from an uninjured lower extremity is the first choice for an interposition graft.
The repair of concomitant venous injuries is a controversial injury. Proximal extremity veins and the
great veins are repaired whenever technically possible to avoid the sequela of venous occlusion. Venous
repair should not be attempted, in a hemodynamically unstable patient; rather, ligation should be
performed to expedite the operation.

28. Penetrating injuries to the pancreas and duodenum are uncommon occurring in 4% and 6% of
patients, respectively. Which of the following statement(s) is/are true concerning the management of
pancreaticoduodenal injuries?

a. The Kocher maneuver is essential for providing exposure for the duodenum

b. A large injury of the duodenum which cannot be closed primarily will always require a
pancreaticoduodenectomy

c. Pyloric exclusion involves suture or staple closure of the pylorus, gastrojejunostomy, tube
decompression of the duodenum, and placement of a T-tube in the common bile duct

d. Class III injuries of the head of the pancreas should be treated with simple external drainage rather
than resection

Answer: a, d

Because of the retroperitoneal location of the duodenum and pancreas and the close proximity to a
number of viscera and major structures, isolated penetrating injuries to the duodenum and pancreas are
rare. Diagnosis of pancreaticoduodenal injuries depends on adequate exposure. A Kocher maneuver
whereby the duodenum and head of the pancreas are mobilized from the retroperitoneal position by
excising the lateral peritoneal reflection of the duodenum is essential for this exposure. Most
penetrating injuries of the duodenum are simple lacerations that can be repaired primarily. Large
injuries to the duodenum are more difficult to repair. Injuries of greater than 50% can lead to luminal
compromise if repaired primarily. Treatment with a jejunal patch or duodenojejunostomy with a
defunctionalized Roux-en-Y limb of jejunum can avoid the need for pancreaticoduodenectomy and its
associated substantial mortality. Since many duodenal repairs are tenuous especially in combination
with pancreatic injury and the concern about the digestive action of activated pancreatic enzymes on
the repair, the technique of pyloric exclusion has been devised and is advocated by some. Pyloric
exclusion involves suture or staple closure of the pylorus and restoration of gastrointestinal continuity
by performing a gastrojejunostomy. Tube decompression of the duodenum should be performed in
severe duodenal injuries but the biliary tract does not require decompression unless there has been an
associated biliary tract injury. The management of pancreatic injuries depends on the location with
respect to the head, body, and tail of the gland. Class III injuries of the head of the pancreas involve a
relatively severe injury. In almost all situations, these injuries should simply be drained without attempts
at resection or emergency internal drainage. If a patient develops a pancreatic fistula, the fistula can be
controlled by the drain. If the fistula does not resolve with time, the pancreas can be drained internally
at a later date.

29. A CT scan is performed on this patient. Which of the following statement(s) is/are true concerning
the findings on CT scan and the patient’s management?

a. The CT finding that correlates most significantly with intracranial hypertension is compression or
obliteration of the basilar cisterns

b. Intracranial pressure monitoring is indicated immediately in any patient with cisternal compression.

c. A brain contusion appears as a very homogeneous high density area in the cerebral cortex

d. Intracerebral hematomas are routinely treated with craniotomy

Answer: a, b

The CT finding which correlates most significantly with intracranial hypertension is compression or
obliteration of the basilar cisterns. Not only does this finding portend a stormy intracranial pressure
course, but the primary predictor of outcome in patients with this CT picture is the peak level
intracranial hypertension occurring during the first 72 hours. Intracranial pressure monitoring should be
immediately initiated in any patient with cisternal compression and the intracranial hypertension should
be vigorously treated. Intracerebral hemorrhage and cerebral contusion are common after trauma and
are readily visualized on CT scan. Brain contusion appears as a focal, heterogeneous density with
hemorrhage interspersed with injured tissue. Intracerebral hematomas are generally more
homogeneous in their high density appearance. These lesions tend to “blossom” over time due to
continued hemorrhage and the development of edema. Therefore, it is important to closely observe and
monitor the ICP of such patients because a significant and hazardous mass effect may evolve, requiring
surgical extirpation. Cerebral contusions and intracerebral hematomas are treated operatively only
when a mass effect results in intracranial hypertension or signs of herniation.

30. Which of the following statement(s) is/are true concerning the management of chest trauma?
a. The majority of injuries to the chest require surgical intervention

b. The posterior lateral thoracotomy is the optimal approach for emergency thoracotomy

c. Either computed tomography or angiography are suitable methods for detecting aortic disruption in a
patient with an abnormal chest x-ray

d. Persistent bleeding associated with a penetrating injury to the chest is often due to injury to an artery
of the systemic circulation

Answer: d

The chest radiograph is by far the most important diagnostic study in patients with chest trauma and
should be obtained early in all patients. Angiography is the best study to rule out major injury to the
great vessels in the chest, and angiography remains mandatory in the majority of patients at risk for
aortic disruption who have an abnormal chest radiograph. At the present time, CT scan of the chest
appears to have a higher rate of missed injury than angiography for assessment of the aorta and should
probably be avoided in patients with abnormal chest films.

The majority of injuries to the chest can be successfully managed without surgical intervention. The
routine use of a tube thoracostomy for treatment of hemothorax and pneumothorax is the cornerstone
of therapy. Thoracotomy is most often needed for the control of massive bleeding, or bleeding which
persists despite tube thoracostomy. About 80% to 85% of hemorrhages within the chest can be treated
by tube thoracotomy alone. Even larger and deep lacerations of the lung parenchyma, which bleed with
relatively low pressure from the pulmonary circulation, will be controlled by the reinflated lung
parenchyma as well as edema in the tissue from the injury. Persistent bleeding is most commonly due to
injuries to major proximal branches of the pulmonary circulation or injuries to systemic arteries
including intercostal arteries and internal mammary arteries. The choice of position and surgical
approach for thoracotomy for thoracic injury is dictated by the nature of the patient’s injuries, the
certainty of diagnosis, and the potential for associated injuries involving other body sites. Although the
standard postero-lateral thoracotomy provides optimal exposure to the contents of a particular
hemithorax, the lateral position of the patient makes access to the other side of the chest or abdomen
difficult if not impossible. Therefore, though postero-lateral thoracotomy provides the best access, it can
be used only in patients who have injuries isolated to a given hemithorax. In most patients undergoing
emergency thoracotomy for chest trauma, an antero-lateral approach must be used in patients supine
to allow access to the abdomen and contralateral chest cavity. Although exposure through this incision
is considerably more difficult, it is adequate with proper technique. Median sternotomy incision
provides excellent exposure to the heart and the great vessels in the anterior mediastinum, but it
provides very difficult exposure for repair of injuries to the lungs, descending aorta, chest wall,
diaphragm, or esophagus. Therefore, like the postero-lateral thoracotomy, it can be used only when the
patient’s injuries can be determined with relative certainty.

31. The anterior neck is divided into three zones defined by horizontal planes. Which of the following
statement(s) is/are true concerning penetrating injuries to the anterior neck?

a. Penetrating injuries to Zone I carry the highest mortality

b. Injuries to Zone II are the most common and the mortality rate is second only to those of Zone I

c. Exposure of Zone III for detection of injuries to the distal carotid artery and pharynx can be quite
difficult

d. All hemodynamically stable patients with penetrating injuries to Zone I should have angiography

e. Most vascular lesions in Zone III are best treated by surgical exploration

Answer: a, c, d

The anterior neck is divided into three zones defined by horizontal planes. Zone I represents the base of
the neck and it invariably extends from the sternal notch to the top of the clavicles or the cricoid
cartilage. Injuries here carry the highest mortality because of the risk of major vascular and intrathoracic
injury. Zone II is the mid-body and largest portion of the neck. It extends from the top of Zone I to the
angle of the mandible. Zone II injuries are most common but carry a lower mortality rate than either
Zone I or Zone III injuries, since the injury is generally apparent and exposure of the vital structures is
readily accomplished. Zone III is that part of the neck above the angle of the mandible. The risk of injury
to the distal carotid artery, salivary glands and pharynx is greatest in this zone. Exposure in this region
can be particularly difficult.

Most surgical groups advocate exploration in the majority of penetrating neck wounds that penetrate
the platysma in Zone II and in all patients with clinical signs of tracheal, esophageal, or major vascular
injury. Preoperative angiography is generally not required for Zone II injuries because of the relative
ease of exposure and control of critical vascular structures. Zone I and III penetrating injuries are
selectively managed based on clinical presentation and the result of diagnostic studies.
Hemodynamically unstable patients are immediately explored with operative incision based on the most
likely source of vascular injury. Zone I injuries are essentially managed similar to mediastinal traversing
wounds. Angiography is performed in all hemodynamically stable patients with penetrating wounds to
Zone I to identify potential injuries to the thoracic outlet vessels or to plan better operative approach.
Angiography is also performed for Zone III injuries, because of the possible inaccessibility of the internal
carotid artery lesions or to demonstrate a need for systemic anticoagulation. Furthermore, most of the
vascular lesions identified at the base of the skull are best managed by interventional angiography
techniques.

32. Which of the following statement(s) is/are true concerning the definitive management of neck
injuries?

a. Patients with evidence of an acute stroke following penetrating injury involving the carotid artery
should be managed with arterial ligation

b. Unilateral vertebral artery occlusion usually results in a clear neurologic deficit and therefore
revascularization is indicated

c. The combination of esophography and endoscopy improves the accuracy of detecting esophageal
injury with penetrating trauma

d. External drainage is an important aspect of the surgical management of an esophageal injury

e. Arterial dissection secondary to blunt trauma is best managed by operative exploration and resection
of the dissection

Answer: c, d

Blood vessels are the most commonly injured structures in the neck. Major arterial injuries occur in 18%
of penetrating neck wounds and major venous injuries in 26%. Blunt vascular injures account for a very
small percentage of carotid injuries, however their management is somewhat controversial with
treatment highly variable dependent upon the vascular lesion as well as concomitant injuries. When
anatomically feasible, pseudoaneurysms are probably best managed by resection. The best treatment
for arterial dissection, however, although not completely resolved, would appear to be systemic
anticoagulation if possible to prevent propagation, embolization or thrombosis. Resection may not be
required in the majority of patients. Penetrating carotid injury most commonly presents with
exsanguinating hemorrhage. The indication for repair versus ligation of a carotid injury depends, in part,
on the neurologic presentation. Patients without a neurologic deficit and a carotid injury should have
restoration of vascular continuity with good neurologic outcome anticipated. Also, patients with all
grades of neurological deficits short of coma should have primary vascular repair. Although experience
with revascularization of patients suffering acute stroke from arteriosclerotic occlusive disease suggests
that hemorrhagic infarction and death may result from revascularization, several reviews of acute
revascularization in the trauma patient note that combined morbidity and mortality are significantly less
in those patients repaired primarily compared to those managed with arterial ligation. Traumatic injury
to the vertebral arteries are now more commonly identified due to the more liberal application of neck
angiography. Unilateral vertebral artery occlusion seldom results in a neurologic deficit. Treatment of
blunt vertebral artery injury with thrombosis generally is nonoperative: systemic anticoagulation is
recommended to avoid further propagation of existing thrombus.

The diagnosis of esophageal injury can be difficult. The sensitivity of esophography in detecting
esophageal injuries varies from 50% to 90%; the sensitivity of endoscopy ranges from 29% to 100%.
These modalities should be considered complimentary, and when combined have an accuracy of nearly
100%. Since virtually all reported deaths from cervical esophageal injuries are the result of delayed or
misdiagnosis, a particularly high index of suspicion is warranted. When injured, the esophagus should be
meticulously debrided and repaired primarily in one or two layers. It is important to drain all such
wounds, because infection or salivary fistula is not an infrequent complication.

33. A 25-year-old male is involved in a motor vehicle accident with a significant head injury. Which of the
following statement(s) is/are true concerning his injury and management?

a. A single episode of systolic blood pressure < 90 mm Hg occurring during the early period after injury
significantly increases the chances of mortality and morbidity

b. Systemic hypertension should be avoided to reduce the risk of intracranial hemorrhage

c. The patient should be vigorously hyperventilated to reduce PaCO2

d. The patient should be heavily sedated and pharmacologically paralyzed after the initial neurologic
examination

Answer: a

Brain injury is the most common cause of death in trauma victims, accounting for about half of deaths at
the accident site. The injuries are generally the result of blunt trauma, and motor vehicle accidents are
the most frequent cause. Head injuries involve not only the primary injury but secondary injuries which
can result from the events occurring after the primary insult, due to either the direct consequences of a
process initiated by the primary injury or to deleterious outside influences. The occurrence and
magnitude of secondary insults is often the determining factor in outcome from brain injury. Since
secondary insults, in contrast to primary injuries, are amenable to medical therapy, they are the focus
toward which the medical treatment of brain injury is directed. The primary external secondary injury
processes occurring following brain injury are hypotension and hypoxia. Hypotension is the number one
treatable determinant of severe head injury. A single episode of systolic blood pressure less than 90 mm
Hg occurring during the period from injury through resuscitation doubles the mortality and significantly
increases the morbidity of any given brain injury. Intracranial hypertension may be considered as being
deleterious via two somewhat separate mechanisms—herniation and ischemia. Herniation occurs when
a pressure gradient exists across an incomplete barrier such as the tentorium or the falx cerebri. It is
deleterious because of the tissue damage that results when herniation occurs. The second aspect of the
intracranial hypertension that is deleterious is elevated resistance to cerebral blood flow, resulting in or
exacerbating ischemia. Treatment of systemic hypertension is rarely indicated in the head injured
patient. There is no evidence that hypertension promotes continued intracranial hemorrhage, and
hypertension related to brain injury generally resolves when the intracranial hypertension is controlled.
The treatment of intracranial hypertension involves elevating the head of the bed (reversed
Trendelenburg position) but should only be performed after complete resuscitation has been
accomplished. The confusion and agitation often attendant to head injury renders sedation desirable,
therefore, patients with suspected head injury should generally be sedated. Pharmacologic relaxation,
however, has the notable effect of limiting the neurologic examination to the pupils and, upon arrival to
the hospital, the computed tomography scan. Therefore, its use in the absence of evidence of herniation
should be limited to situations which sedation alone is not sufficient to optimize safe and efficient
patient transport and resuscitation. When used, short acting agents are strongly preferred. Prophylactic
administration of mannitol is not recommended due to volume depleting diuretic effect. In addition,
although it is desirable to approximate the lower end of the normal range of PaCO2 during transport of
a patient with suspected brain injury, the risk of exacerbating early ischemia by vigorous
hyperventilation outweighs the questionable benefit in the patient without evidence of herniation.
Therefore, ventilatory parameters consistent with optimal oxygenation and “normal” ventilation are
recommended.

34. Which of the following statement(s) is/are true concerning the biomechanics of penetrating injuries?

a. Stab wounds are associated with significant cavitation

b. A hollow point bullet is associated with an enlarged area of injury

c. A high velocity gunshot wound creates a vacuum pulling clothing, bacteria, and other debris into the
wound

d. The frontal area of impact of a bullet is determined by the caliber of the bullet

Answer: b, c
Penetrating trauma involves the transfer of energy to a relative small tissue area. The kinetic energy of a
bullet disrupts and fragments cells and tissues, moving them away from the path of the bullet. The
actual size of the frontal area of impact is determined by three factors—profile, tumble (spin and yaw),
and fragmentation. A knife or jacketed bullet does not deform significantly during impact, whereas a
hollow-point bullet flattens, spreads, and fragments on impact and therefore enlarges the area of injury.
Low energy missiles including knives and other objects produce damage only by sharp cutting edges.
Cavitation is minimal, and injury can be predicted simply by tracing the pathway of the weapon within
the body. Low, medium and high velocity gunshot wounds, however, produce damage not only to tissue
directly in the path of the missile but also produce cavitation injury to tissues in close proximity to the
impact. The size of the cavitation injury is directly proportional to the bullet’s velocity. The essential
difference between high velocity weapons and low and medium velocity weapons is that the higher
velocity weapons have a much larger cavity or pressure cone than low-and medium-velocity missiles.
The temporary cavity extends well beyond the actual bullet tract, producing a wider injury. The vacuum
created by the cavitation pulls clothing, bacteria, and other debris from the surrounding areas into the
wound, creating the additional risk of contamination.

35. In which of the following clinical situations is peritoneal lavage indicated?

a. A patient with suspected intraabdominal injury who will undergo prolonged general anesthesia for
another injury outside the abdomen

b. A patient with a high velocity abdominal gunshot wound

c. A patient with an abdominal knife wound

d. A hemodynamically unstable patient with a high suspicion of intraabdominal hemorrhage

e. A patient with major noncontiguous injuries (i.e., chest and lower extremity)

Answer: a, c, e

Peritoneal lavage is a standard technique to detect significant intraabdominal hemorrhage after blunt
trauma. Its applicability after low-velocity gunshot or stab wounds is less clear, but it has no place in the
evaluation of high-velocity gunshot wounds. Abdominal paracentesis can be used in place of peritoneal
lavage when the suspicion of intraabdominal hemorrhage is high and time is critical. Specific indications
for peritoneal lavage and blunt trauma include a number of conditions such as a patient with major
noncontiguous injuries, a patient with suspected intraabdominal injury in whom physical examination is
unreliable or impossible due to the need for prolonged general anesthesia for another injury. Peritoneal
lavage is not useful for patients with abdominal gunshot wounds; all of these patients require
immediate laparotomy. When local examination of a stab wound suggests penetration to the anterior
fascia and peritoneum, diagnostic peritoneal lavage may help discriminate between those with
significant and insignificant injuries.

36. Physiologic responses to hypothermia include:

a. Tachycardia regardless of core temperature

b. Tachypnea regardless of core temperature

c. Pupillary dilatation and loss of cerebral autoregulation at temperatures below 26°C

d. A cardiac rhythm contraindicates cardiopulmonary resuscitation even in the absence of a palpable


pulse

Answer: c, d

The physiologic response to hypothermia is one of transitional changes, with few exact temperature-
dependent responses. Broadly speaking, the transition from a “safe zone” of hypothermia (where
physiologic adaptations to heat loss are working) to a “danger zone” of hypothermia occurs between
33°C and 30°C. The cardiovascular response includes tachycardia followed by progressive bradycardia
which starts at about 34°C , and which results in a 50% heart rate decrease at 28°C. Asystole occurs
below 25°. Due to difficulty in palpating weak, bradycardic pulses in cold, stiff hypothermic patients, the
presence of an organized rhythm should be taken as a sign of life that contraindicates CPR, despite the
absence of a palpable pulse. Respiratory drive is increased during the early stages of hypothermia, but
below 30°C progressive respiratory depression occurs, resulting in a decrease in minute ventilation.

The neurologic response to hypothermia is heralded by progressive loss of lucidity and deep tendon
reflexes, and eventually flaccid muscular tone. Pupillary dilatation and loss of cerebral autoregulation
occur at temperatures below 26°, and electroencephalography becomes silent at 19–20°. It is important,
however, to remember that patients have been revived with core temperatures as low as 17°C, and
therefore the saying “No one is dead until warm and dead”.

37. Which of the following statement(s) is/are true concerning the injury pattern in patients with blunt
versus penetrating injuries?
a. Solid organs are most frequently injured following blunt trauma

b. The liver is the most frequently injured organ in both penetrating and blunt trauma

c. Major vascular injuries occur much more commonly in penetrating trauma than with blunt abdominal
trauma

d. Injury patterns for blunt abdominal trauma in children are different than adults whereas with
penetrating trauma no such difference exists

Answer: a, c, d

Most series list the spleen as the most commonly injured intraabdominal organ after blunt trauma.
However, the means of diagnosis may affect this finding since small liver injuries, often detected only on
CT scan of the abdomen, may go unreported while splenic injuries are likely to be clinically significant
and require surgical intervention. Solid organs are most frequently injured from blunt trauma since the
sudden application of pressure to the abdomen is more likely to rupture a solid organ than a hollow
viscus, and this accounts for the greater incidence of solid organ injury. More elastic tissues of young
people tolerate trauma better than those of older people, and this accounts, in part, for the differences
in injuries between children and adults with blunt abdominal trauma. Major vascular injuries occur in
over 10% of patients sustaining penetrating trauma but occur in only approximately 2% of patients with
blunt trauma.

38. An 18-year-old male suffers a gunshot wound to the abdomen, resulting in multiple injuries to the
small bowel and colon. Which of the following statement(s) is/are true concerning this patient’s
perioperative management?

a. A multi-agent antibiotic regimen is indicated

b. Antibiotics should be continued postoperatively for at least 7 days

c. Laparotomy, as a diagnostic test for postoperative sepsis, should be considered

d. The incidence of postoperative wound or intraabdominal infection would be increased in association


with a colon injury

Answer: d
Post-traumatic intraabdominal infection is almost always the result of gastrointestinal tract
contamination. Penetrating trauma accounts for the largest proportion of these infections. Because of
the higher bacterial counts, the colon is consistently associated with a higher incidence of infectious
complications than isolated gastric, duodenal, or small bowel injuries. The precise incidence of
intraabdominal or incisional wound infection after colonic injuries depends on factors present at the
time of injury (blood loss, degree of contamination, and other associated injuries) and on whether the
wound is closed or left open. The use of perioperative antibiotics for trauma has been investigated
extensively. Most studies have demonstrated that single-agent cephalosporins are at least as effective
as multi-agent regimens in retarding intraabdominal abscess or wound infections resulting from a
variety of contaminated traumatic wounds.

Fever, leukocytosis, tachycardia, the development of a paralytic ileus, increased fluid requirements, and
failure to wean from a mechanical ventilation may all represent warning signs of the development of
intraabdominal infection. CT is the single most useful diagnostic tool in this clinical setting because it
yields considerable information with regard to organ injury in the presence of intraabdominal abscesses
or fluid collections. Laparotomy, as a diagnostic tool for unexplained sepsis, has a low-yield in critically ill
trauma patients and should not be used routinely.

39. A middle-aged man is undergoing laparotomy for blunt abdominal trauma. The spleen and liver are
both found to be injured. Which of the following statement(s) is/are true concerning the management
of these injuries?

a. If the patient has multiple other abdominal injuries and hypotension, splenic salvage should not be
attempted

b. The incidence of life-threatening sepsis in the adult following splenectomy is no greater than in the
normal population

c. All liver injuries regardless of their depth require external drainage

d. The Pringle maneuver should control all bleeding from hepatic parenchymal vessels

e. If concern for a biliary fistula from the liver parenchyma exists, a T-tube should be placed even if the
common bile duct is otherwise normal

Answer: a
Solid abdominal organs such as the liver and spleen, are most commonly injured during blunt abdominal
trauma. The management of splenic trauma has been the subject of major reexamination in the last few
decades. Historically, splenic injuries are routinely treated with splenectomy. With increased
appreciation of the danger of post-splenectomy sepsis, splenic salvage procedures and nonoperative
management of these injuries have become well accepted. This is particularly true in children. The
incidence of post-splenectomy sepsis varies from 0.5% to as much as 12% to 15%, depending on the age
and underlying disease. The incidence is inversely related to age and is higher with underlying
hematologic disorders such as lymphoma or thalassemia. The incidence of life-threatening sepsis in
adult trauma patients is low, but higher than in the normal population. Splenic salvage should not be
attempted if the patient has protracted hypotension or other severe injuries or if undue delays are
encountered in an attempt to repair the spleen.

Simple lacerations of the liver found at the time of surgery do not require drainage unless they are deep
into the liver parenchyma, in which case they have a high probability of postoperative bile leakage.
Biliary fistulas usually will close spontaneously, and major extrahepatic ductal injuries are rare. A T-tube
placed in an otherwise normal common bile duct is inappropriate unless the extrahepatic biliary tree is
injured. In the event that bleeding continues despite segmental ligation of parenchymal vessels, the
structures of the porta hepatis should be compressed as a diagnostic maneuver (Pringle maneuver). If
the bleeding stops, it is assumed to originate from the portal veins or hepatic artery. If the bleeding
continues, it is presumed to arise principally from the hepatic veins or inferior vena cava.

40. Which of the following conclusions can be drawn from prospective randomized studies involving
restoration of circulation in the field?

a. Pneumatic anti-shock garment is of benefit only in patients with a field blood pressure less than 50

b. Patients with major vascular injury should not receive intravenous fluid infusion until bleeding can be
controlled in the operating room

c. Hypertonic saline, used as resuscitation fluid, provides no benefit to patients

d. Hypertonic saline has been shown to exacerbate bleeding and precipitate coagulopathy

Answer: a, b

The most common cause of death during the first hour after injury is hemorrhage. Therefore, after
establishment of patent airway and adequate air exchange, the next priority is to support the
circulation. The standard of care in the pre-hospital setting for hypotensive patients has been volume
replacement and application of pneumatic anti-shock garment. In a recent large prospective randomized
study, pneumatic anti-shock garments offered no survival advantage and actually increased mortality
when used in patients with thoracic injuries. On the other hand, there was the suggestion that patients
with a field blood pressure less than 50 mm Hg may benefit from this treatment. A second prospective
study confirmed this result, indicating that the pneumatic anti-shock garment is of value to selected
patients with field blood pressure less than 50 mm Hg. A recent clinical study has also demonstrated
that internal hemorrhage from major vascular injuries should not be treated with intravenous fluid
infusion until bleeding can be controlled in the operating room. In the hypotensive state, such major
vascular injuries have a chance to clot and temporarily stop bleeding. But if intravenous volume restores
blood pressure, the clot may dislodge and the rate of bleeding significantly increases. This may lead to
both loss of oxygen carrying capacity and clotting factors, and ultimately exsanguination. Hypertonic
saline restores intravascular volume and blood pressure to near normal very rapidly. The prospective
randomized trial of normal saline versus hypertonic saline demonstrated a significant improvement in
survival when the data were normalized to a select group of patients. There was no evidence that
nontamponaded bleeding was exacerbated by the use of hypertonic saline despite the fact that blood
pressure and intravascular volume increased.

41. Which of the following statement(s) is/are true concerning hypothermia following traumatic injury?

a. The majority of patients presented to a level I trauma center are hypothermic at some time

b. The initial temperature for trauma-associated hypothermia is associated with no seasonal variation

c. Moderate levels of hypothermia (34°–32°C) has no effect on mortality in the trauma patient

d. The coagulation system is most affected in hypothermic patients who have sustained major trauma

Answer: a, b, d

Mild hypothermia is very common following traumatic injury and is considered a form of secondary
accidental hypothermia. It has been reported that 57% of trauma patients admitted to a level I trauma
center are hypothermic at some time, with temperature loss most significant in the Emergency Room.
This effect appears to have no seasonal variation. Although the mortality rate for moderate (28°–32°C)
degrees of primary accidental hypothermia is only approximately 21%, the same level of hypothermia in
surgical patients who are victims of trauma can be associated with mortality rates approaching 100%.
Hypothermia affects multiple systems, however, the system most affected in patients sustaining major
trauma are those involved in clotting. This effect seems to involve both abnormalities in platelet and
clotting function.

42. Which of the following statement(s) is/are true concerning injuries to the chest wall?

a. The mortality rate currently associated with sternal fractures is as high as 25–30%

b. The severe ventilatory insufficiency associated with a flail chest is due to the paradoxical motion of
the involved segment of chest wall

c. In most cases of an open pneumothorax, or sucking chest wound, surgical closure is necessary

d. Persistent chest tube bleeding at a rate greater than 200 ml/hour for four hours, or greater than 100
ml/hour for eight hours is an indication for thoracotomy for control of hemorrhage

e. A 20% incidence of splenic injury is associated with fractures of ribs 9, 10 and 11 on the left

Answer: c, d, e

Rib fracture is the most common injury associated with blunt chest trauma and may occur directly at the
site of force or laterally as the result of significant antero-posterior compression of the chest. The
location area of the rib fracture may be indicative of associated injuries. A 20% incidence of splenic
injury is associated with fracture of ribs 9, 10, and 11 on the left with a similar association with right
lower rib fractures and hepatic parenchymal injuries. The mortality rate associated with sternal fractures
in older series was as high as 25–30%, mainly because of other injuries to the chest, such as aortic
transection, cardiac contusion, tamponade or tracheo-bronchial rupture. More recent studies have
suggested a change in the pattern and severity of injuries associated with sternal fracture. Widespread
improvements in automobile safety have likely contributed to this change such that isolated sternal
fractures may result from shoulder belt use and may not necessitate hospital admission in the stable
patient. A flail chest occurs when consecutive ribs are fractured in more than one place, creating a free-
floating segment of the chest wall. The creation of a free-floating segment may result in paradoxical
chest wall motion with respiration. The intact chest wall expands during inspiration, but the negative
intrathoracic pressure generated causes the flail segment to move inappropriately inward. Historically it
was believed that the paradoxical motion was the cause of severe ventilatory insufficiency associated
with the flail chest. Gradually, understanding of the pathophysiology of the flail chest has evolved. The
ventilatory impairment is not simply due to paradoxical motion of the chest wall, but rather due to
underlying pulmonary parenchymal injury in combination with the hypoventilation and splinting that
results from the pain of multiple contiguous rib fractures. The open pneumothorax, or sucking chest
wound, is an uncommon injury usually caused by impalement, high-speed motor vehicle accident, or
shotgun blast, which causes a large chest wall defect. The diagnosis of a sucking chest wound can be
made on simple inspection of the chest wall and hearing the flow of air through the wound. The defect
should be occluded immediately with an impermeable dressing, essentially converting the situation to a
closed pneumothorax. Tube thoracostomy is then performed to re-expand the lung. The chest wall
defect usually requires operative debridement and formal chest wall closure. A hemothorax is the
accumulation of blood in the pleural space and it occurs in 50–75% of patients with severe blunt or
penetrating chest trauma. Massive hemothorax (i.e., larger than 1000–1500 ml) may require
thoracotomy. Persistent bleeding, at a rate of > 200 ml/hour for four hours, or > 100 ml/hour for eight
hours, is also an indication for thoracotomy. If the patient manifests any hemodynamic instability during
the period of observation, urgent thoracotomy is mandatory.

43. A 22-year-old male driving a car at a high speed and not wearing a seatbelt, leaves a road and
crashes with a full frontal impact into a tree. Which of the following injury patterns may be predictable
from this type of motor vehicle accident?

a. Orthopedic injuries involving the knees, femurs, or hips

b. Laceration to the aorta

c. Hyperextension of the neck with cervical spine injury

d. Diaphragmatic rupture due to marked increase in intraabdominal pressure

Answer: a, b, c

With frontal impact, when the vehicle stops abruptly, unrestrained front-seat occupants move in one of
two predictable pathways—down and under the dashboard or up and over the steering wheel. With the
former movement, the knees strike the dashboard, and the upper legs absorb the primary energy
transfer. Dislocated knees, fractured femurs, and posterior fracture dislocation of the hips are expected
injuries. After the knees impact, the upper body flexes forward and up and over the steering wheel. The
chest or abdomen impacts the steering wheel and the head impacts the wind shield.

Predictable injury patterns following the up-and-over component of a frontal impact include the
following: 1) anterior chest wall compression; 2) compression injuries to both hollow and solid
abdominal viscera; 3) shear injuries such as lacerations to the aorta or liver, kidneys or other solid
viscera; 4) injury to the brain from direct compression with scalp lacerations, skull fractures and cerebral
contusions or from deceleration or shear forces; 5) acute neck flexion, hyperextension or both resulting
in cervical spine injury.

Three-point passenger restraints and air bags, although overall very effective in reducing injury, can
cause specific related injuries. Common injuries when lap belts are incorrectly strapped above the
anterior iliac spine include compression injuries of intraabdominal organs (liver, pancreas, spleen, small
bowel, large bowel), increased intraabdominal pressure and diaphragmatic rupture.

44. Which of the following statement(s) is/are correct concerning the pathophysiology of frostbite?

a. Frostbite injury may have two components: initial freeze injury and a reperfusion injury that follows
during rewarming

b. The formation of extracellular ice crystals in the tissue begins to occur at -10°C

c. The release of oxygen free radicals and arachidonic acid metabolites aggravates vasoconstriction and
platelet and leukocyte aggregation

d. Experimental evidence suggests that a substantial component of severe cold injury may be mediated
due to platelet aggregation

Answer: a, c

Recent evidence suggests that frostbite injury may have two components: the initial freeze injury, and a
reperfusion injury that occurs during rewarming. The initial response to tissue cooling is vasoconstriction
and arterio-venous shunting, intermittently relieved by vasodilatation. With prolonged exposure, this
response fails, and the temperature of the freezing tissues will approximate ambient temperature until -
2°C. At this point, extracellular ice crystals form, and as these crystals enlarge, the osmotic pressure of
the interstitium increases resulting in movement of intracellular water into the interstitium. Cells begin
to shrink and become hyperosmolar, disrupting cellular enzyme function.

During rewarming, red cell, platelet and leukocyte aggregation is known to occur and results in patchy
thrombosis of the microcirculation. These accumulated blood elements are thought to release, among
other products, the toxic oxygen-free radicals and the arachidonic acid metabolites which further
aggravate vasoconstriction and platelet and leukocyte aggregation. Recent experimental evidence
suggests that a substantial component of severe cold injury may be neutrophil-mediated in that a
monoclonal antibody to neutrophil-endothelial and neutrophil-neutrophil adherence can markedly
ameliorate the pathology of severe injury.
45. The management of a patient with frostbite includes:

a. Gradual spontaneous warming

b. Emersion of the tissue in a large water bath with a temperature of 40–42°C

c. Immediate initiation of prophylactic antibiotics

d. Systemic anticoagulation with heparin

e. Immediate debridement of necrotic tissue

Answer: b

The treatment of frostbite with rewarming should begin in the Emergency Room and not in the field.
Gradual, spontaneous warming is generally inadequate and delayed thawing, or rubbing the injured part
in ice or snow often results in marked tissue loss. Rapid rewarming should be achieved by immersing the
tissue in a large bath of 40–42°C. The water should feel warm, but not hot to the normal hand. The skin
should be gently but meticulously cleansed, air dried, and affected area elevated to minimize edema.
Infection develops in only about 13% of urban frostbite victims, but half of these infections are present
at the time of admission. Therefore, most clinicians reserve antibiotics for identified infections.
Following rewarming, the treatment goals are to prevent further injury while awaiting demarcation of
the irreversible tissue destruction. The use of sympathetic blockade, surgical sympathectomy, and
intraarterial vasodilating drugs has generally been ineffective. Heparin, thrombolytic agents, and
hyperbaric oxygen have also failed to demonstrate any substantial treatment benefit. The difficulty in
determining the depth of tissue injury and cold injury has led to a conservative approach to the care of
frostbite injuries. As a general rule, amputation and surgical debridement are delayed for 2–3 months
unless infection with sepsis intervenes. The natural history of full thickness frostbite is gradual
demarcation of the injured area with dry gangrene and mummification clearly delineating a nonviable
tissue.

46. There are a number of injuries associated with common orthopedic injuries. Which of the following
diagnosed orthopedic injuries is associated with the injury listed?
a. Sternal fracture—cardiac contusion

b. Posterior dislocation of the knee—popliteal artery thrombosis

c. Pelvic fracture—ruptured bladder or urethral transection

d. Posterior dislocation of hip—-sciatic nerve injury

Answer: a, b, c, d

PATTERNS OF INJURY TO THE HEAD, NECK, TRUNK, AND EXTREMITIES ASSOCIATED WITH ORTHOPEDIC
INJURIES

Diagnosed Injury Associated Injury

Fracture—temporal, parietal bone Epidural hematoma

Maxillofacial fracture Cervical spine fracture

Sternal fracture Cardiac contusion

First and second rib fracture Descending thoracic aorta, intraabdominal bleeding

Fractured scapula Pulmonary contusion

Fractured ribs 8–12, right Lacerated liver

Fractured ribs 8–12, left Lacerated spleen

Fractured pelvis Ruptured bladder, urethral transection

Fractured humerus Radial nerve injury

Supracondylar humerus Brachial artery injury

Distal radius fracture Median nerve compression

Supracondylar femur fracture Thrombosis popliteal artery

Anterior dislocation shoulder Axillary nerve injury

Posterior dislocation of hip Sciatic nerve injury

Posterior dislocation of knee Popliteal artery thrombosis


47. Correct statement(s) concerning cold injury include:

a. Chilblain is a form of local cold injury characterized by pruritic papules, macules, or plaques on the
skin associated with repeated exposure to cold temperatures

b. Trenchfoot is a freeze injury of the hands or feet due to chronic exposure to cold, wet conditions
below freezing

c. Frost nip is reversible with warming of the tissue and will result in the return of sensation and function
with no tissue loss

d. Characteristic large blisters can be seen with all degrees of frostbite

Answer: a, c

Cold injuries limited to digits, extremities, or exposed surfaces are the result of either direct tissue
freezing (frostbite) or more chronic exposure to an environment just above freezing (Chilblain or pernio;
trenchfoot). Chilblain or pernio are descriptive forms of local cold injury characterized by pruritic, red-
purple papules, macules, plaques or nodules in the skin. This pathology appears to be provoked by
repeated exposure to cold but not freezing temperatures. Trenchfoot or cold emersion foot describes a
non-freezing injury of the hands or feet, typically in sailors, fishermen, or soldiers resulting from chronic
exposure to wet conditions and temperatures just above freezing. Frost nip is the mildest form of cold
injury characterized by initial pain, pallor, and subsequent numbness of the affected body part. The
injury is reversible and warming of the cold tissue results in return of sensation and function with no
tissue loss. Frostbite is more severe and common form of cold injury and essentially describes local
freezing of tissues. The mildest form (first degree injury) is associated with hyperemia and edema but
without blistering. Second, third and fourth degree frostbite have progressive degrees of tissue injury
and are noted by either characteristic clear blisters (second degree) or more hemorrhagic vesicles which
are generally smaller than second degree blisters (third degree frostbite). In fourth degree frostbite,
tissue necrosis, gangrene and full thickness tissue loss can be seen.

48. A 37-year-old man driving an automobile travelling at a rapid speed hits a tree. At arrival to the
Trauma Center, aortic disruption is suspected. Which of the following statement(s) is/are true
concerning the patient’s diagnosis and management?

a. If undiagnosed, a thoracic aortic disruption is associated with a 50% mortality within the first 24 hours
b. Transesophageal echocardiography is a promising new modality for the diagnosis of aortic injury

c. Repair of aortic disruption is best completed with cardiopulmonary bypass

d. Pharmacologic control of blood pressure with sodium nitroprusside should be used routinely in the
preoperative management

Answer: a, b

Blunt injuries to the thoracic aorta occur in as many as 20% fatalities due to motor vehicle accidents.
About half of these patients die at the scene. It is estimated that of the 50% who survive the initial
injury, half will die within the first 24 hours and 90% will die within 10 weeks without surgical treatment.
Blunt aortic disruption is associated with the mechanism of abrupt deceleration. Therefore, this
mechanism of injury should lead to high index of suspicion. A chest radiograph is a useful screening
procedure. Abnormal findings on chest film, or suspicion of the injury, must be aggressively investigated.
Due to the very high morbidity of missed injuries, angiography is the diagnostic study of choice in
patients at significant risk. Transesophageal echocardiography is a promising modality for the diagnosis
of aortic injury, especially in patients who cannot be transported to the angiography suite. Early
experience has shown transesophageal echo to be a very sensitive method, with very few missed
injuries in experienced hands.

Injuries to the aorta require surgical repair. The technique of aortic repair has been the subject of some
controversy primarily due to the risk of spinal cord ischemia with cross clamping of the thoracic aorta.
The complete use of cardiopulmonary bypass with full heparinization, however, has been shown to
increase the mortality of patients who have other cerebral and vascular injuries, and is probably
contraindicated in the blunt trauma patient. Most surgeons favor cross clamping of the aorta with
expeditious repair of the injury. Rapid surgical repair is vital to survival of the patients. Preoperative
management of patients with aortic disruption involves careful control of blood pressure and avoidance
of hypertension. Pharmacologic control of blood pressure is indicated to avoid possible rupture before
surgical repair. The use of sodium nitroprusside, however, should be avoided in patients with head
injuries because the vasodilatory effect of this drug may cause an increase in intracranial pressure. A
short-acting beta agonist such as esmolol or labetolol is probably the best choice if blood pressure
control is needed.

49. Which of the following statement(s) is/are true concerning endotracheal intubation at the site of
injury?
a. Bag valve mask systems are equally as efficient as endotracheal intubation for early management of
the trauma patient

b. Paramedic intubation in the field is successful in over 90% of cases

c. Indications for intubation in the field include respiratory distress, significant head injury, severe chest
injury and hypovolemic shock

d. If patients clench their teeth violently, endotracheal intubation is impossible without the use of
paralytic agents

Answer: b, c

The most immediately life-threatening problem to the injured patient is loss of airway patency and
therefore this is the first priority of the first response team upon arrival at the injury site. Basic life
support skills such as suctioning, placement of oropharyngeal airways, the use of a bag mask device are
usually sufficient at least to temporarily restore oxygenation at the injury site. On the other hand,
approximately 10% of patients require endotracheal intubation and up to 20% would benefit from field
intubation. Endotracheal intubation is the best procedure for airway control in patients who are in
shock, have abnormal breathing patterns, or who are unable to protect their airways due to
unconsciousness. Endotracheal intubation is far superior than that of bag valve mask systems because it
provides larger total volumes and less risk of aspiration.

Indications for endotracheal intubation in the field should include respiratory distress, hypovolemic
shock, unconsciousness, significant head injury, and severe chest injury. Reported paramedic intubation
success rates range between 90 and 98% in the literature, and complications are rare. On the other
hand, there are problems with intubation at pre-hospital sites. Patients with head injuries may have C-
spine injuries so in-line mobilization techniques are necessary to insure intubation without further injury
to the cervical spine or cord. Patients often clench their teeth in which case either nasotracheal
intubation or the use of paralytic agents such as succinylcholine may be necessary for successful
intubation.

50. Which of the following statement(s) concerning the operative approach to abdominal trauma is/are
correct?

a. Pelvic hematomas associated with pelvic fractures should be explored


b. Central retroperitoneal hematomas should be explored after control of other injuries within the
peritoneal cavity

c. Stable hematomas in the perinephric space lateral to the midline should be explored to rule out renal
injury

d. The initial approach is control of hemorrhage by packing and controlling ongoing contamination from
enteric injuries

Answer: b, d

Once the abdomen is opened at laparotomy for trauma, obvious blood and clot is sequentially removed,
first from the lower abdomen and then from the upper abdomen by packing all four quadrants of the
abdomen. Any areas found to be a source of hemorrhage can be repacked. Obvious hollow viscus
wounds should be rapidly sutured or controlled with noncrushing clamps. Once hemorrhage is
controlled by packing and ongoing contamination is stopped, time is then taken to allow resuscitation of
the patient’s circulating blood volume. Retroperitoneal hematomas may be the source of exsanguinated
hemorrhage if rupture into the free peritoneal cavity has occurred. If not, these can be left for
investigation at a later time, depending on the location. Hematomas of the pelvis that are associated
with pelvic fractures should not be disturbed. Similarly, stable hematomas of the perinephric space
lateral to the midline are also best left undisturbed. Central hematomas that may involve injuries to the
major vascular structures, pancreas or duodenum are noted and explored after control of injuries within
the peritoneal cavity.

51. Which of the following statement(s) is/are true concerning trauma involving children?

a. The greater head/body ratio in children compared to adults leads to a higher frequency of head
injuries in children

b. Unfused cranial sutures and open fontanels serve as a protective mechanism against intracranial
hemorrhage

c. A greater propensity to hypothermia is seen in children

d. A propensity to single organ system injury is seen in the child

Answer: a, c
The smaller size of pediatric patients results in an increased likelihood of multiple system trauma
because of the force of impact is dissipated over a relatively small area. A higher frequency of head
injuries in children is partially explained by the proportionately greater head/body ratio, the thin skull,
and the weaker supporting cervical musculature. In infants with unfused cranial sutures and open
fontanels, intracranial hemorrhage can be perfuse and result in shock. The protuberant abdomen of the
child obtains little protection from either the thoracic cage or pelvis, accounting for a higher incidence of
intraabdominal injuries.

The physiologic response to hypovolemia after pediatric trauma is characterized by the immediate
constriction of small and medium-sized arteries, thus maintaining normal blood pressure.
Decompensation generally occurs with a blood volume deficit of 20% to 25%. Tachycardia, tachypnea,
diminished peripheral perfusion, and change in the level of consciousness are better potential indicators
of early shock than blood pressure. The thin skin, lack of subcutaneous fat, and large surface area/body
weight ratio all contribute to the propensity of the young child for hypothermia.

52. Indications for Cesarean section during laparotomy for trauma include:

a. Maternal shock after 28 weeks gestation

b. Unstable thoracolumbar spinal injury

c. Mechanical limitation for maternal repair

d. Maternal death if estimated gestational age is at least 28 weeks

nswer: b, c, d

The indications for exploratory laparotomy in a pregnant patient are the same as in all other trauma
patients. However, Cesarean section should not be added unless indicated due to the prolongation of
operative time and the increase in blood loss (approximately 1 liter). Vaginal delivery is always
encouraged even in the postoperative period. During laparotomy for trauma, indications for Cesarean
section are as follows:

1. Maternal shock, pregnancy near term

2. Threat to life from exsanguination

3. Mechanical limitation for maternal repair


4. Risk of fetal distress exceeding risk of prematurity

5. Unstable thoracolumbar spinal injury

The outcome of postmortem C-section depends on the duration of the gestation and the time interval
between maternal death and delivery. Under optimal conditions, at 26 to 28 weeks gestation, estimated
fetal survival is about 50%. Post-mortem C-section is justified if the estimated age is about 26—28
weeks. If the time interval between maternal death and delivery is less than 5 minutes, the fetal
prognosis is considered excellent. If the time interval since maternal death is prolonged to about 20
minutes, fetal prognosis is poor.

53. A 75-year-old man is involved in a motor vehicle accident. Which of the following statement(s) is/are
true concerning this patient’s injury and management?

a. Acceptable vital sign parameters are similar across all age groups

b. Hypertonic solutions should not be used for resuscitation due to concerns for fluid overload

c. The patient would be more prone to a subdural hematoma than a younger patient

d. There is no role for inotropic agents in the management of this patient

Answer: c

Although most principles of management of the elderly trauma patient are similar to their younger
counterpart, some important differences must be noted. Evaluating the circulatory system following
injury in the elderly, it must be remembered that elderly patients most likely are accustomed to a higher
than normal blood pressure. Thus, while a systolic blood pressure of 100 mm Hg is not alarming in a 25
year old, in a 75 year old, this may very well represent hypotensive shock if the “normal” pressure is 150
mm Hg systolic. Recent reports have suggested that pulmonary arterial catheters can be useful in the
monitoring of patients with evidence of shock or hypoperfusion or history of intercurrent disease. In
patients with a low pulmonary capillary wedge pressure, volume replacement can be provided as
needed, however, in the face of an elevated pulmonary capillary wedge pressure, inotropic support may
be of benefit. Lactated Ringer’s solution remains the resuscitation fluid of choice in the elderly patient.
However, the initial experience with hypertonic solutions have been very favorable. Hypertonic fluids
can reduce elevated blood pressures and improve cardiac performance with much smaller volumes
when compared to isotonic solutions.
Cerebral atrophy accompanies aging. In addition, the cerebral vasculature is fragile, particularly the
veins. The combination of these factors make the elderly more prone to develop subdural hematomas,
which may initially be subtle.

54. Important physiologic alterations of pregnancy which may alter the injury response include:

a. Increased cardiac output

b. Expanded plasma volume

c. Decreased fibrinogen and clotting factors

d. Partial obstruction of the inferior vena cava

Answer: a, b, d

55. A number of systems have been developed in an effort to allow comparison of trauma injuries and
trauma patients among institutions. Which of the following statement(s) is/are true concerning trauma
scoring systems?

a. The Revised Trauma Score uses the physiologic parameters of blood pressure, heart rate, and head
injury to mathematically assess injury severity

b. The Abbreviated Injury Scale (AIS) is a specific anatomic index

c. The Injury Severity Score (ISS) correlates not only the severity of the injury but adjusts for patient age
and comorbid risk factors

d. The Triss System is the most complete system in combining trauma score and anatomic component as
well as patient age

Answer: a, b, d

Many systems have been developed in an effort to allow comparison of trauma injuries and trauma
patients among institutions. The impetus for injury severity scoring system is provided by the need to
identify and classify severely injured patients in the pre-hospital phase, to predict mortality, to assess
results, and to improve communication. The Revised Trauma Score has been the most widely applied as
well as the most useful scoring system for the initial evaluation of trauma victims. It assumes that the
physiologic parameters of blood pressure, respiratory rate, and head injury (assessed by the Glasgow
Coma Score) can be used mathematically to assess injury severity and predict the most timely and
sophisticated medical care. The Abbreviated Injury Scale (AIS), initially devised for blunt trauma and
subsequently updated to include penetrating trauma, assesses the severity of nonfatal injuries
determined in six different body areas. Thus, it is a specific anatomic index. The Injury Severity Score
(ISS) is calculated by assigning the AIS values to each injury in six body parts and then mathematically
squaring the three most severely injured areas and adding the total. Unfortunately, this system does not
adjust for patient age or patient-related comorbid risk factors. The Triss methodology is of great
importance because it attempts to combine the trauma score, or physiologic component, and the ISS, or
anatomic component. It also incorporates the patient’s age. The Triss method yields a specific
probability of survival, and is recommended for use by the American College of Surgeons Committee on
Trauma to be used to maintain a trauma registry and quality assurance program.

56. Alterations in the immunologic response after a major trauma include:

a. Decreased CD3 and CD4 population

b. Depression of neutrophil antimicrobial functions including chemotaxis and phagocytosis

c. Decreased levels of pro-inflammatory cytokines including tumor necrosis factor, interleukin-1, and
interleukin-6

d. Impaired macrophage receptor expression and antigen presentation

Answer: a, b, d

Major perturbations in the immune system occur in patients after injury contributing to the late septic
mortality in trauma patients. The changes in the immune system are significant and global, affecting
both humoral and cellular components of the system. Macrophage receptor expression and subsequent
antigen presentation are impaired with similar defects in lymphocyte function including shifts in T-cell
populations with decreased CD3 and CD4 subpopulations, depression of B-cell and immunoglobulin
production, and a loss of antigen recall. Multiple neutrophil antimicrobial functions are suppressed
following trauma including chemotaxis, phagocytosis, respiratory bursts, and intracellular killing.
There are significant changes in humoral mediators following trauma with increased levels of pro-
inflammatory cytokines including tumor necrosis factor, interleukin-1 and interleukin-6 along with
decreased levels of interleukin-2, interleukin-3, and interferon g.

57. Which of the following statement(s) is/are true concerning penetrating injuries to the colon and
rectum?

a. A patient with 2 or more additional organs injured, significant fecal spillage, preoperative
hypotension, or intraperitoneal hemorrhage exceeding 1 liter should not have a primary repair of a
colon injury

b. If rectal injury is documented, a loop colostomy provides adequate decompression.

c. Irrigation of the rectal stump should be avoided to prevent contamination via the site of injury

d. The rectal wall should be repaired in all cases

Answer: a

The central issue in the operative management of colonic injuries is the controversy between primary
repair of low-risk colonic injuries and repair or resection with exteriorization. Primary repair may be
selected when additional risk factors have been excluded. Complications increase with primary repair
when there is preoperative hypotension, intraperitoneal hemorrhage exceeding 1 liter, more than two
additional organs injured, significant fecal spillage, or when more than six hours have elapsed since
injury. Many patients with low-risk penetrating colon injuries can be treated with primary closure in the
absence of these risk factors. High-risk colon injuries are those associated with severe injuries, as
indicated above, and should be treated with resection and colostomy.

Rectal injuries should be suspected when there is any penetrating injury or a significant pelvic fracture.
Sigmoidoscopic examination is essential. The principles of operative management include wide
debridement of all dead and devitalized tissue and repair of rectal wall when possible. A totally
defunctioning colostomy (not a simple loop colostomy) is necessary. Retrorectal drainage is indicated
only in selected severe injuries. The distal stump should be washed out to evacuate the fecal contents.
Broad-spectrum intravenous antibiotics, nutritional support, and serial debridements are also indicated.
58. Genitourinary injuries are common with both blunt and penetrating trauma. Which of the following
statement(s) is/are true concerning genitourinary trauma injuries?

a. All patients with microscopic hematuria and blunt trauma should be evaluated with an intravenous
pyelogram

b. The indications for radiographic assessment of renal injury in the face of blunt trauma is more liberal
than penetrating trauma

c. CT scan is the current imaging technique of choice for suspected renal trauma

d. Perinephric hematomas occurring after either penetrating or blunt trauma should not be explored

e. Extraperitoneal bladder ruptures can often be treated nonoperatively using urethral catheter
drainage alone

Answer: c, e

Renal injuries constitute the greatest proportion of genitourinary tract trauma. The presence of
hematuria remains the most sensitive clinical indicator of renal trauma. The specificity of hematuria is
low, however, and the practice of performing an IVP in all patients with blunt trauma and microscopic
hematuria is both time-consuming and unnecessary. In several studies examining clinical features
associated with significant renal trauma, three factors have been identified—shock, gross hematuria,
and major associated injuries. The incidence of renal trauma requiring operation in the absence of any
of these factors was 0 in several series. The indications for radiographic assessment of renal injury in the
face of penetrating trauma should be far more liberal, since there are conflicting reports on the degree
of correlation between the injury’s severity and the degree of hematuria. Radiographic studies for the
diagnosis of renal trauma include single-or multiple-film IVP, formal nephrotomography, and CT scan.
Single-film (“one-shot”) IVP is useful primarily for documenting the presence of two functioning kidneys
and has limited use as a screening examination for renal trauma. CT scan, however, has emerged as the
imaging technique of choice for most renal trauma. Renal injuries can be staged with respect to those
likely to require an operation or to develop complications. CT scan also allows more precise assessment
of the degree of perinephric hemorrhage and the degree of collecting system disruption than operative
inspection.

A number of major renal injuries are diagnosed at the time of laparotomy. Most commonly, a
perinephric hematoma is encountered in association with blunt hepatic or splenic trauma. Indications
for renal exploration at laparotomy following blunt trauma include an expanding or pulsatile perinephric
hematoma or suspected renal vascular injury. In a patient with blunt injuries, it is preferable to defer
exploration of nonexpanding, nonpulsatile perinephric hematomas to complete treatment of
intraabdominal and other associated life-threatening injuries. Postoperative CT scan may be useful for
formal staging of these injuries. A perinephric hematoma that is found during laparotomy for
penetrating trauma should generally be explored carefully. Unlike blunt injuries, continued or recurrent
hemorrhage is more often a problem.

Most bladder injuries (over 95%) occur in association with pelvic fractures. Bladder ruptures are
classified into those that rupture into the free peritoneal cavity and those with extravasation limited to
the retroperitoneum. Intraperitoneal bladder ruptures are characteristically large and require early
operative repair. Extraperitoneal bladder ruptures in most cases, however, can be treated
nonoperatively using simple urethral catheter drainage alone.

59. In children who sustain multiple trauma, 25% have serious intraabdominal injuries. Which of the
following statement(s) is/are true concerning blunt abdominal trauma in children?

a. Peritoneal lavage plays an important role in the evaluation of the patient

b. Most pediatric trauma patients will be hemodynamically unstable at the time of admission

c. Splenic salvage can be achieved in 90% to 100% of patients

d. The indications for laparotomy for splenic injury include refractory hypotension or transfusion
requirement in excess of 50% of blood volume within the first 24 hours

e. Unlike splenic injury, hepatic injury will frequently require exploratory laparotomy

Answer: c, d

Diagnostic peritoneal lavage is a rapid and sensitive test for the presence of intraabdominal
hemorrhage. In general, however, peritoneal lavage has a limited role in the care of pediatric patients as
it provides confirmation of a finding, hemoperitoneum, that does not mandate operation. Unlike adults,
95% of pediatric trauma patients are hemodynamically stable on admission. The single most useful
diagnostic maneuver is a CT scan, which delineates solid visceral injuries with a high degree of both
sensitivity and specificity. The spleen and liver are injured with about equal frequency in children
sustaining blunt trauma, and together these two target organs account for about 75% of childhood
abdominal injuries.

Experience at virtually every major children’s trauma center in the world supports the safety and
efficacy of nonoperative management of children with splenic ruptures. Most series report splenic
salvage in 90% to 100% of children. Although therapy must be individualized, the general guidelines are
that operation is not indicated until there is refractory hypotension or a transfusion requirement in
excess of 50% of blood volume in the first 24 hours. In reality, few patients approach this, and
transfusion practices are such that only 10% to 20% of patients with isolated splenic injuries require
blood transfusion at all. The management of liver injuries in pediatric patients has also changed since
the advent of routine abdominal CT scan for blunt trauma. Several reports describe successful
nonoperative treatment of liver injuries detected radiologically in children. This approach is applicable in
most children, with a success rate of 90% and transfusion requirements similar to those patients with
ruptured spleens.

60. Which of the following statement(s) is/are true concerning the diagnosis and management of pelvic
fractures secondary to blunt trauma?

a. Most pelvic fractures are apparent on the basis of physical examination

b. An infra-umbilical approach to peritoneal lavage in a patient with a major pelvic fracture may yield a
false-positive rate approaching 50%

c. If a large expanding pelvic hematoma is found at surgery, the intraabdominal injury should be dealt
with, and the hematoma explored

d. The application of pelvic external fixation may be used as the initial step in control of hemorrhage
from pelvic fractures

e. A urethral catheter should be placed immediately in patients with suspected pelvic fracture to allow
early peritoneal lavage

Answer: b, d

The spectrum of pelvic fracture injuries range from minor isolated non-displaced fractures of the pubic
rami to severe injuries with multiple fractures that can be rapidly lethal. Unlike most long bone
fractures, only 25% of pelvic fractures are apparent on physical examination. Hemorrhage caused by
laceration of the sacral venous plexus, multiple arterial branches of the hypogastric vessels, or bleeding
from fractured cancellous bone presents a formidable challenge to the trauma surgeon. Massive
hemorrhage is the principle cause of early death in patients with pelvic fracture, and survival depends
principally on rapid identification and control. The presence of hemorrhage from associated
intraperitoneal injuries should be considered first, therefore diagnostic peritoneal lavage is indicated for
most patients with pelvic fractures. A supraumbilical lavage is preferable under these circumstances
because the possibility of catheter penetration of a large retroperitoneal hematoma dissecting into the
preperitoneal space. Peritoneal lavage performed incorrectly in the infraumbilical site with a major
pelvic fracture may yield an incidence of false-positive results as high as 45%. When performed properly
in the supraumbilical position, false-positive lavage rates have been reported to be as low as 1%. If
laparotomy is indicated once a thorough abdominal exploration is performed and injury is repaired, the
size of the pelvic hematoma may be assessed. If a rapidly expanding pelvic hematoma is seen, rapid
closure of the abdominal wound is indicated followed immediately by pelvic angiography and
embolization of active arterial bleeding. In selected patients with unstable fractures involving the
sacrum or pubic diastasis injuries, the application of pelvic external fixation may reduce hemorrhage
from cancellous bone and sacral venous plexus. In many centers, pelvic fixation is preferred to
arteriography and embolization for the initial control of bleeding.

A urethral tear should be suspected in any male with a pelvic fracture. These patients should be
examined carefully for signs of urethral injury including scrotal or perineal hematoma, blood at the
urethral meatus, or anterior displacement of the prostate gland on rectal examination. The presence of
any of these clinical findings constitutes a contraindication to immediate placement of a urethral
catheter. A retrograde urethrogram should be obtained in these cases by the placement of a small
balloon catheter in the fossa navicularis and gravity infusion of 10–15 ml of contrast medium.

61. Which of the following statement(s) is/are true concerning the Advanced Trauma Life Support (ATLS)
classification system of hemorrhagic shock?

a. Class I shock is equivalent to voluntary blood donation

b. In Class II shock there will be evidence of change in vital signs with tachycardia, tachypnea and a
significant decrease in systolic blood pressure

c. Class III hemorrhage can usually be managed by simple administration of crystalloid solution

d. Class IV hemorrhage involves loss of over 40% of blood volume loss and can be classified as life-
threatening

Answer: a, d

The classification of hemorrhagic shock as defined by the ATLS classification system of the American
College of Surgeons is useful in comprehending the manifestations and physiologic changes associated
with shock. Mild hemorrhage, up to 15% of total blood volume, is exemplified by voluntary blood
donation. In the supine position, there are no measurable changes in heart or respiratory rates, blood
pressure or pulse pressure. Class II hemorrhage involves loss of 15% to 30% of volume loss. Clinical signs
include tachycardia and tachypnea. The systolic blood pressure may be only slightly decreased,
especially in the supine position, but the pulse pressure is narrowed. Patients with Class II hemorrhage
can generally be resuscitated with crystalloid solutions but some may require blood transfusions. With
Class III hemorrhage, 30% to 40% of total body volume is lost. Patients with Class III hemorrhage present
with obviously inadequate perfusion; marked tachycardia and tachypnea, cool, clammy extremities with
significant delayed capillary refill; hypotension; and significant changes in mental status. Class III
hemorrhage represents the smallest volume of blood loss that consistently produces a decrease in
systolic blood pressure. The resuscitation of these patients requires blood transfusion in addition to
crystalloid administration. Class IV hemorrhage involves loss of greater than 40% of blood volume. This
represents life-threatening hemorrhage. These patients require immediate transfusion for resuscitation
and frequently require immediate surgical intervention.

62. Which of the following statement(s) is/are true concerning traumatic pericardial tamponade?

a. The condition only develops in cases of penetrating trauma

b. Beck’s triad, consisting of muffled heart sounds, decreased pulse pressure, and jugular venous
distention can be seen in most patients

c. Two-dimensional echocardiography has replaced diagnostic pericardiocentesis in most


hemodynamically stable patients

d. The majority of patients with a small injury to a single chamber of the heart arriving with vital signs at
the hospital will die of their injuries

Answer: c

Injuries to the heart resulting in cardiac tamponade can occur from either blunt or penetrating trauma,
though penetrating injuries are much more common. Pericardial tamponade occurring after blunt
trauma usually results from rupture of a chamber of the heart, with many associated with death at the
scene. Penetrating trauma is the usual cause of pericardial tamponade and the outcome is directly
related to the character of the weapon. Reported survival rates for small injuries to a single chamber are
between 60 and 87%, although patients who arrive moribund do poorly regardless of care. The diagnosis
of pericardial tamponade should be considered in any patient with penetrating chest trauma,
particularly to the central portion of the chest. The classic Beck’s triad, consisting of muffled heart
sounds, decreased pulse pressure, and jugular venous distention, occurs in only the minority of patients.
The diagnosis of pericardial tamponade can be somewhat difficult but should be suspected in patients
with trauma who remain hypotensive and have no evidence of external blood loss or hemorrhage into
the thorax, abdomen, or pelvis. Unfortunately, CVP measurements are neither sensitive or specific for
the diagnosis of pericardial tamponade, and are dependent upon the patient’s volume status and the
level of agitation. Two-dimensional echocardiography is very sensitive to the presence of pericardial
fluid and wall motion abnormalities. If available in a timely fashion, cardiac echo is a very good
diagnostic test to rule out cardiac tamponade in a stable patient. Under most circumstances, there is no
role for diagnostic pericardiocentesis.

63. The intravenous fluid that a 60 kg., 30-year-old woman with an 80% burn should be given in the first
24 hours following burn injury is:

A. 19.2 liters of 5% glucose in lactated Ringer's.

B. 14.4 liters of lactated Ringer's.

C. 9.6 liters of hypertonic salt solution (sodium concentration 200 mEq. per liter).

D. 7.2 liters of 5% albumin solution.

E. 5.5 liters of the pentafraction component of hydroxyethyl starch.

Answer: B

DISCUSSION: The consensus range for estimating fluid needs of burn patients in the first 24 hours is 2 to
4 ml. of a physiologic crystalloid solution per kilogram body weight per percent of body surface burned.
In this patient that would be 9.6 to 19.2 liters of lactated Ringer's solution. The early elevation of
circulating levels of catecholamines and glucocorticoids following burn injury induces glycogenolysis,
which results in elevated circulating blood glucose levels. Glucose should not be administered in the
resuscitation fluids, since the resulting exaggeration of hyperglycemia could induce osmotic diuresis. In
the first 24 hours colloid-containing solution is not commonly used, but if it is used even the Evans
formula recommends only 1 ml. per kg. body weight per percent of body surface burned. Moreover,
patients in one study who received colloid-containing fluids continued to gain weight during the first 3
postinjury days, retained more sodium, and had less urine output than patients who received only
crystalloid fluids in the first 24 hours. Hypertonic salt solution is also not commonly used for burn
patient resuscitation because of the recently described and surprisingly frequent occurrence of acute
renal failure and increased mortality associated with its use. If hypertonic salt is used, the amount
infused should be less than 9 liters, to avoid excessive elevation of the serum sodium concentration (i.e.,
above 160 mEq. per liter). The recommended limit of hydroxyethyl starch infusion is currently 1500 ml.
per day. Although a 10% pentastarch form of hydroxyethyl starch has been used to expand the plasma
volume of burn patients at the end of the first 24 hours, even as little as 500 ml. of that solution has
been reported to prolong both prothrombin and plasma thromboplastin time.
64. Indications for escharotomy of a circumferentially burned right lower limb include all of the
following except:

A. Progressively severe deep tissue pain.

B. Coolness of the unburned skin of the toes of the right foot.

C. A pressure of 40 mm. Hg in the anterior compartment of the distal right leg.

D. Edema of the unburned skin of the right foot.

E. Absence of pulsatile flow in the posterior tibial artery.

Answer: BD

DISCUSSION: The blood flow to distal and underlying unburned tissue in a limb can be compromised by
vascular compression due to edema formation beneath the unyielding eschar of a full-thickness
circumferential burn. The most reliably noninvasive means of monitoring adequacy of the circulation in
a circumferentially burned limb is serial examination using the ultrasonic flowmeter. The absence or
progressive diminution of pulsatile flow in the posterior tibial artery in the lower limb or the palmar arch
arteries in the upper limb indicates a need for escharotomy. Delayed capillary refilling, cyanosis of the
digits, and progressively severe paresthesias, particularly deep tissue pain, are all clinical signs that may
indicate vascular compromise and should be monitored if an ultrasonic flowmeter is not available.
Persistent deep tissue pain and progressively severe paresthesias are the most reliable of the
nonspecific clinical signs. A muscle compartment pressure that exceeds 30 mm. Hg, which is greater
than normal capillary pressure, has also been used as an indication for escharotomy in burn patients.
Edema and coolness to the touch of distal unburned tissue commonly accompany thermal injury and are
not useful in assessing the need for escharotomy.

65. Which of the following is/are true about inhalation injury in burn patients?

A. A chest x-ray obtained within 24 hours of injury is an accurate means of diagnosis.

B. Its presence characteristically necessitates administration of resuscitation fluids in excess of


estimated volume.

C. When moderate or severe, it exerts a comorbid effect that is related to both extent of burn and the
age of the patient.
D. It increases the prevalence of bronchopneumonia.

E. Prophylactic high-frequency ventilation reduces the occurrence of pneumonia and the mortality in
burn patients with inhalation injury.

Answer: BCDE

DISCUSSION: Extensive inflammatory changes are evoked in the airway following the inhalation of
smoke and other irritating products of incomplete combustion. Clinical signs are nonspecific and may be
delayed. Chest x-rays are also unreliable in detecting even severe inhalation injury. Chest x-rays taken
within 24 hours of injury were found to be falsely negative in 92% of 106 patients with inhalation injury.
Fiberoptic bronchoscopic examination is the most reliable single means of diagnosing inhalation injury
of the large airways, but in patients who inhaled finely particulate smoke the large airways may show
little if any inflammatory change. The distance smoke particles travel before deposition in the airways is
inversely related to particle size. When the smoke particle mass median diameter is less than 0.5 mm.,
deposition occurs in the terminal bronchioles and alveoli. In such patients, inhalation injury is best
identified by prolonged retention of xenon 133 as assessed by a ventilation perfusion lung scan.

Historically, fluid restriction was recommended for patients with inhalation injury, but in recent years it
has become obvious that such patients typically require more resuscitation fluid than the volume
estimated by commonly used formulas. Edema of the small airways and occlusion due to endobronchial
sloughing and inspissation predispose burn patients to develop pneumonia. In one study, 46% of burn
patients with inhalation injury developed pneumonia and 69% of the pneumonias occurred in the first
postburn week. The comorbid effect of moderate to severe inhalation injury is related to both age and
burn size and increases mortality by a maximum of 20% above that predicted on the basis of age and
burn size in patients whose burn injury alone would be associated with a 75% likelihood of death. In
patients with only mild inhalation injury there is little if any increase in mortality above that predicted
on the basis of age and burn size alone. Prophylactic use of high-frequency percussive ventilation
minimizes airway collapse and atelectatic changes, as a consequence of which the incidence of
pneumonia is reduced and survival is significantly increased.

66. Adequacy of fluid resuscitation in burn patients is indicated by which of the following?

A. Urine output of 45 ml. per hr. in a 70-kg. 30-year-old man with flame burns involving 55% of the total
body surface.

B. Hourly urine output of 7 ml. in a 7-kg. 15-month-old child with burns involving 40% of the total body
surface.
C. A pulmonary capillary wedge pressure of 17 to 20 mm. Hg.

D. Hourly output of 40 ml. of port wine–colored urine in an 80-kg. male who has severe high-voltage
electric injury of the right arm and left leg.

E. A urinary sodium concentration of 4 mEq. per liter.

Answer: ABC

DISCUSSION: The goal of burn patient resuscitation is the maintenance of vital organ function at the
least immediate or delayed physiologic cost. Fluid resuscitation of burn patients does not need to be a
test of maximum renal function. Adequacy of volume replacement and of renal blood flow are indicated
by an hourly urine output of 30 to 50 ml. in adults and 1 ml. per kg. per hr. in children weighing less than
30 kg. In patients with extensive muscle damage caused by high-voltage electric injury, heavy loads of
hemochromagens give the urine the appearance of port wine. Such patients are prone to develop acute
renal failure unless brisk urine output is maintained until the pigment concentration is reduced to
insignificant levels. Fluid should be infused into such patients at the rate needed to achieve an hourly
urine output of 75 to 100 ml., but if the patient does not respond to increased fluid input with an
increase in urine volume and clearing of the heme pigments, a diuretic should be given. A pulmonary
capillary wedge pressure of 17 to 20 mm. Hg is indicative of an adequate circulating blood volume, but a
urinary sodium concentration of less than 20 mEq. per liter is consistent with an intravascular volume
deficit.

67. Common electrolyte changes during and after resuscitation in a patient with a burn of 65% of the
total body surface include:

A. A serum sodium concentration of 128 mEq. per liter following 48 hours of resuscitation fluid therapy.

B. A serum sodium concentration of 152 mEq. per liter on the fifth postburn day in a 75-kg. male with a
75% burn who has received only calculated maintenance fluids each day following successful
resuscitation.

C. A serum potassium concentration of 5.7 mEq. per liter as a consequence of the destruction of red
cells and other tissues in a patient with high-voltage electrical injury.

D. Hypokalemia due to the kaliuretic effect of 0.5% silver nitrate soaks.

E. Hypocalcemia with a low ionized calcium level on the third postburn day as a consequence of dilution
and hypoalbuminemia.

Answer: ABC
DISCUSSION: At the end of the first 48 hours of resuscitation, when lactated Ringer's solution is used in
the first 24 hours and colloid-containing fluid and electrolyte-free fluid in the second 24 hours, modest
hyponatremia (serum sodium concentration of 128 to 130 mEq. per liter) is commonly observed but
requires no treatment. The total body salt load is actually increased, and appropriate fluid management
permits the increased evaporative water loss to correct that imbalance. The most common
postresuscitation fluid and electrolyte disturbance is hypernatremia associated with dehydration due to
inadequate replacement of insensible water loss. The hourly insensible water loss, which far exceeds
maintenance fluid requirements in uninjured patients, can be calculated thus:

Insensible water loss (in ml./hr.)=(25 + % of body surface burned) × total body surface area (sq. m.)

The release of potassium from red cells and other tissues injured by the burn or by electrical current can
cause usually modest hyperkalemia. If acidosis occurs, the hyperkalemia may be exaggerated to
symptomatic levels that require treatment. Hypokalemia can be induced following resuscitation by the
kaliuretic effect of sulfamylon burn cream, but the hypokalemia associated with 0.5% silver nitrate soak
treatment is due to transeschar leaching of potassium. Hypocalcemia is frequently associated with
hypoalbuminemia as a consequence of hemodilution by the resuscitation fluid and the cytokine-induced
reprogramming of hepatic protein synthesis. In such cases ionized calcium levels are commonly normal.

68. The clinical and histologic signs of invasive burn wound infection include which of the following?

A. Focal dark red or dark brown discoloration of the eschar.

B. Delayed separation of the eschar.

C. Conversion of an area of partial-thickness burn to full-thickness necrosis.

D. The presence of micro-organisms in the unburned subcutaneous tissue in a burn wound biopsy
specimen.

E. Perineural and perivascular microbial migration through the eschar with proliferation of micro-
organisms in the subeschar space.

Answer: ACD

DISCUSSION: It is essential to examine the entire burn wound at the time of the daily cleansing to
identify invasive burn wound infection at the earliest possible time. The appearance of focal areas of
dark red or dark brown discoloration are the most common changes indicative of burn wound infection,
but similar changes may be caused by hemorrhage due to local trauma or maceration. Accelerated
separation of the eschar is often produced by burn wound infections, but delayed separation of the
eschar is indicative of effective control of the microbial population in the burn wound. Conversion of an
area of partial-thickness burn to full-thickness necrosis is the most reliable clinical sign of invasive burn
wound infection. Identification of such a change mandates histologic examination of a burn wound
biopsy, which is the only reliable means of differentiating the colonization of nonviable tissue from the
invasion of viable tissue. Identification of micro-organisms in the unburned viable tissue of a burn
wound biopsy confirms the diagnosis of invasive burn wound infection. Microbial migration along the
skin appendages, terminal nerve radicles, and thrombosed capillaries in the eschar and heavy growth of
micro-organisms in the subeschar space are manifestations of the colonization of nonviable tissue and
represent the mechanisms by which eschar separation occurs.

69. The treatment of invasive burn wound infection may include which of the following?

A. Subeschar infusion of half the daily dose of a broad-spectrum penicillin suspended in 1 liter of normal
saline.

B. Use of 0.5% silver nitrate soaks for topical therapy.

C. Specific systemic antibiotic therapy.

D. Excision and immediate autografting.

E. Amputation when the infection has extended to involve underlying muscle.

Answer: ACE

DISCUSSION: The Pseudomonas organisms that most commonly cause invasive bacterial burn wound
infection are typically sensitive to high concentrations of broad-spectrum penicillins. When the diagnosis
of invasive Pseudomonas burn wound infection has been made, one half of the daily dose of a broad-
spectrum penicillin, typically piperacillin, suspended in 1 liter of normal saline, should be infused into
the subeschar tissues beneath the infected wound. A number 20 spinal needle should be used for the
infusion, to minimize the number of injection sites. Following a second subeschar infusion of the broad-
spectrum penicillin just prior to operation, the infected tissue should be excised. The excised wounds
should not be autografted but covered with a biologic dressing or a dressing soaked with an
antimicrobial solution such as 5% mafenide acetate. The patient is returned to the operating room in 24
to 48 hours to examine the excised wound and assess the adequacy of the débridement. The frequency
of perilymphatic and perivascular proliferation of invading Pseudomonas organisms is associated with a
risk of metastatic spread to remote organs or tissues. Consequently, systemic antimicrobial therapy
should be instituted based on the sensitivity patterns of the resident microbial flora and adjusted as
necessary on the basis of the patient's culture and sensitivity results. Amputation is frequently necessary
to control invasive burn wound infection when a phycomycotic infection on a limb has traversed the
investing fascia and involves significant amounts of the subfascial tissue.

70. The treatment of patients with high-voltage electric injury differs from that of patients with
conventional thermal injury with respect to the need for:

A. Fasciotomy.

B. Hemodialysis.

C. Amputation.

D. Pulse oximetry.

E. Prehospital cardiopulmonary resuscitation.

Answer: ABCE

DISCUSSION: Both lightning injury and contact with electric current can induce cardiopulmonary arrest
due to either asystole or fibrillation. Cardiopulmonary resuscitation must be initiated at the site of injury
if cardiac arrest is present. Cardiac arrhythmias may also occur following resuscitation, necessitating
electrocardiographic (ECG) monitoring for at least 48 hours following injury in patients with a history of
loss of consciousness or an abnormal ECG. Tissue damage and tissue destruction beneath the investing
fascia can result in the formation of edema that increases muscle compartment pressure to a level that
necessitates fasciotomy (> 30 mm. Hg). The current flow in a limb in contact with high-voltage current
can be so great as to damage even the periosseous muscles and make amputation necessary. Liberation
of hemochromogens as a consequence of deep tissue injury is associated with an increased incidence of
acute renal failure necessitating hemodialysis. Electric injury does not influence the need for monitoring
by pulse oximetry, and in fact the destruction of deep tissue in a limb may preclude application of the
pulse oximeter to that extremity.

71. Therapeutic interventions needed for specific chemical agents include which of the following?

A. Prolonged saline irrigation of eyes injured by concentrated sodium hydroxide using a scleral lens with
an irrigating sidearm.
B. Administration of an emetic agent as immediate treatment following lye ingestion.

C. Intra-arterial infusion of calcium gluconate for relief of refractory deep tissue pain due to hydrofluoric
acid injury.

D. Use of propylene glycol to remove residual phenol following water lavage.

E. Application of 5% copper sulfate solution soaks to areas of embedded particles of white phosphorus.

Answer: ACD

DISCUSSION: An eye injured by a strong chemical agent must be irrigated immediately at the site of the
accident to minimize corneal damage. Prolonged irrigation for 12 to 72 hours is recommended for eyes
injured by a strong alkali solution. Irrigation is difficult because of blepharospasm unless a modified
scleral contact lens with an irrigating sidearm is used. Emetics should be avoided in the early treatment
of patients following chemical ingestion since additional injury of the esophagus, oropharynx, and upper
airway may be caused as the chemical is regurgitated. The intra-arterial infusion of calcium gluconate
has been reported to limit tissue damage and relieve pain, but local excision of the involved tissue may
be necessary for definitive control of pain and removal of the injured tissue.

Even though phenol is only slightly soluble in water, initial water lavage of burns caused by phenol
should be carried out. Following that initial lavage, the involved area should be washed with a lipophilic
solvent such as polyethylene glycol, propylene glycol, or glycerol to remove the residual phenol. A dilute
0.5% to 1% solution of copper sulfate can be used as a wash to facilitate identification and impede the
ignition of embedded phosphorus particles. If excessive amounts of copper sulfate are absorbed
through the injured tissues, intravascular hemolysis can occur and may cause renal failure.
Consequently, one should avoid the use of more concentrated solutions of copper sulfate and should
never apply the copper sulfate solution as a soak. The important principle of treatment is to prevent
ignition of the particles by preventing their exposure to air, and that can be done most safely by
applying an occlusive dressing soaked with saline or water.

72. Characteristics of the hypermetabolic response to burn injury include:

A. Elevation of core temperature, skin temperature, and core-to-skin heat transfer.

B. Ambient temperature dependency of metabolic rate.

C. A marked increase of blood flow to the burn wound.

D. A curvilinear relationship to the extent of burn.


E. Oxidation of stored lipid as the major source of metabolic energy.

Answer: ACE

DISCUSSION: At thermal neutral and higher temperatures, the core temperature, skin temperature, and
core-to-skin heat transfer in burn patients remain elevated, but metabolic rate can be diminished in
patients with burns of more than 50% of the body surface by maintaining the ambient temperature
above 30‫ ؛‬C. Blocking evaporative water loss by application of an impermeable membrane is not
attended by a consistent diminution in metabolic rate, indicating that the burn patient is not externally
cold but is internally warm. The hypermetabolism in burn patients is temperature sensitive but not
temperature dependent. Even though earlier measurements described a curvilinear relationship
between metabolic rate and extent of burn, recent measurements have shown that metabolic rate
increases in linear fashion and rises to levels of twice normal in patients with burns of 75% and more of
the total body surface. Lipid stores are the major source of metabolic fuel that is oxidized for energy,
and not lean body mass, which undergoes proteolysis to provide the amino acids necessary for protein
synthesis and wound healing as well as gluconeogenic processes that provide fuel for tissues requiring
glucose. Blood flow to a burned limb is markedly increased as compared with flow in an unburned limb
of the same patient, and the flow increase is directed to the wound per se, not the underlying muscles.

73. A 32-year-old mountain climber who struck his head in a fall lay in the snow overnight before he
could be rescued and brought to the hospital. Upon admission he is semicomatose and not shivering,
with a pulse rate of 48 beats per minute and a blood pressure of 80/50 mm. Hg. His rectal temperature
as measured by a standard thermometer is reported as 34‫ ؛‬C. All the digits on both feet appear to be
frozen. Treatment for this patient should include:

A. Administration of inotropic and chronotropic vasoactive agents.

B. Intra-arterial infusion of vasodilating agents.

C. Infusion of lactated Ringer's solution warmed to 40‫ ؛‬C.

D. Immersion in a circulating water bath heated to 40‫ ؛‬C.

E. Excision of damaged tissue within 48 hours after thawing.

Answer: CD

DISCUSSION: A standard clinical thermometer will not measure body temperatures below 34‫ ؛‬C. This
patient's clinical condition—depressed mental status, bradycardia, and hypotension—indicates that the
patient is likely suffering from severe hypothermia and, so, requires prompt active rewarming by
immersion in a water bath at 40‫ ؛‬C. or the use of partial cardiopulmonary bypass, if available, and the
administration of resuscitation fluid warmed to 40‫ ؛‬C. Vasoactive agents do not treat the basic
pathology in hypothermic patients and are typically ineffective. Vasodilating agents will be of little if any
value in the treatment of the frozen tissue in the feet since histologic studies have indicated that the
vasculature in freeze-injured tissue is dilated, not constricted. Since assessment of tissue viability
immediately after thawing is difficult and often erroneous, one should await clear demarcation of dead
tissue before undertaking surgical excision of damaged tissue.

74. Valid points in the management of burns on special areas include:

a. The large majority of genital burns are best managed by immediate excision and autografting

b. All digits with deep dermal and full-thickness burns should be immobilized with six weeks of axial
Kirschner wire fixation

c. Deep thermal burns of the central face are best managed with immediate excision and autografting

d. Burns of the external ear are commonly complicated by acute suppurative chondritis if topical
mafenide acetate is not applied

Answer: d

Because of the thickness and deep appendages of the skin of the central face, relatively deep burns of
these areas frequently heal. This is fortunate, because it is difficult to achieve a favorable result with
primary excision and grafting of the central face. Management of the burned hand is dictated by the
depth of injury. Superficial burns are managed with elevation, topical antimicrobials, and full passive
range of motion for each joint twice daily. Deep, partial and full-thickness injuries are best managed by
excision and sheet grafting as soon as practical. Hands are immobilized in a functional position for seven
days after surgery before passive and active therapy is resumed. Fourth degree hand burns, which
involve the underlying extensor mechanism, joint capsules or bone are significantly more difficult
management problems and are managed by staged sheet autografting and often benefit from
temporary axial Kirschner wire fixation of open and unstable interphalangeal or metacarpophalangeal
joints. Burns of the external ear are treated with twice daily cleansing and application of mafenide
acetate. Deep burns of the external ear are commonly complicated by acute suppurative chondritis if
topical mafenide acetate is not applied. In general, the practice for deep genital burns is to manage
these limited surface area injuries with topical therapy for a period of two to three weeks unless the
wounds are remarkably deep. Unhealed injuries are debrided and grafted with sheet autograft at this
time, with generally excellent cosmetic and functional results.
75. The hypermetabolic response seen in patients with large burns, who are successfully resuscitated, is
thought to be driven by which of the following factors?

a. Deficient gastrointestinal barrier function

b. Bacterial contamination of the burn wound

c. Evaporative heat loss

d. Changes in hypothalamic function

Answer: a, b, c, d

The physiologic challenge of a burn in excess of 20% of the body surface frequently results in an initial
decrease in cardiac output and metabolic rate. Subsequently, effected by a complex cascade of
mediators, a hypermetabolic response is seen with a near doubling of cardiac output and resting energy
expenditure over the next 24 to 48 hours in those who are successfully resuscitated. The magnitude of
this response peaks in those with injuries of 60% or more of the body surface at as high as twice the
normal basal metabolic rate. The etiology of the hypermetabolic response is not entirely understood but
is assumed to involve a combination of factors including a change in hypothalamic function with
coincident increases in glucagon, cortisol and catecholamine secretion, deficient gastrointestinal barrier
function with translocation of bacteria and their byproducts, bacterial contamination of the burn wound
with systemic release of similar products from this source, and some element of enhanced heat loss via
transeschar evaporation of fluid. An important element of successful management of patients who have
sustained large injuries is support of this response through the provision of adequate quantity and
quality of substrate.

76. Which of the following statement(s) is/are true concerning inhalation injury?

a. The physiology of these injuries include upper airway obstruction secondary to progressive edema,
reactive bronchospasm from aerosolized irritants, and microatelectasis from loss of surfactant and
alveolar edema
b. Endotracheal intubation is indicated immediately in all patients with suspected inhalation injury

c. Distal airway injuries are usually caused by heat injury

d. Peak inspiratory pressures of > 40 cm of water are indicated to maintain functional residual capacity

Answer: a

The pathophysiology of inhalation injury is complex and varies with the aerosolized toxins particular to
the circumstances of individual injuries. However, these injuries routinely demonstrate the following: 1)
upper airway obstruction secondary to progressive edema; 2) reactive bronchospasm from aerosolized
irritants; 3) small airway occlusion initially from edema and subsequently from sloughed endotracheal
debris and loss of ciliary clearance mechanisms; 4) microatelectasis from the loss of surfactant and
alveolar edema; and 5) interstitial and alveolar edema secondary to loss of capillary integrity. The
physiologic consequences of these aberrations are upper and lower airway obstruction, increased
airway resistance, decreased compliance, and an increase in the dead space to tidal volume ratio and
intrapulmonary shunting.

Upper airway obstruction is best managed with prompt endotracheal intubation which is maintained for
48 to 72 hours and elevation the head. In equivocal cases, bronchoscopy is performed and patients with
significant airway edema are intubated using the bronchoscope as a stylet. Although severe steam
inhalation can result in direct heat injury to the distal tracheobronchial tree, more distal airway injuries
are usually caused by aerosolized toxins rather than thermal injury, as the upper airway is a highly
effective heat sink. Although moderate inflating pressures will help expand recruitable segments, peak
inspiratory pressures in excess of 40 cm H2O should be avoided because they are associated with both
overt barotrauma as well as more subtle overpressure injuries to the pulmonary microvasculature and
alveoli which themselves exacerbate respiratory failure. High inflating pressures are also ineffective in
recruiting additional lung, because the compliance decrements are not homogeneous and high
pressures simply over distended more compliant segments.

77. Which of the following statement(s) is/are true concerning the initial fluid resuscitation of a burn
patient?

a. Rigid adherence to the Modified Brooke formula is advised

b. In general, children require less fluid than that predicted by standard formulae

c. Patients with inhalation injuries require less fluid than predicted by standard formulae
d. Dextrose should not be given as the primary resuscitative fluid for any age group

e. Most resuscitative formulae withhold colloid solutions until 24 hours post-injury

Answer: e

The large number of fluid resuscitation formulae in common use is attributed to the fact that no formula
accurately predicts fluid requirements in every patient. No formula can replace a physician at the
bedside repeatedly evaluating the patient’s physiology through the resuscitative period. A reasonable
consensus formula is the Modified Brooke formula, however, regardless of the formula chosen to
initiate resuscitation, subsequent fluid administration is best guided by regular assessment of the
resuscitation end points, rather than prediction of any formula. Vasoactive mediators released from the
injured tissue result in diffuse capillary leaks seen shortly after major burn injury with resulting
extravasation of both crystalloid and colloid for the first 18 to 24 hours after burn. The pathophysiology
explains the enormous volume requirements seen in such patients and is the reason that most
resuscitative formulae withhold colloid until 24 hours post-injury. Children have been found to
commonly require fluid in excess of that predicted by several formulae. These requirements are
generated if one uses a urine output of 1–2 cc/kg/hour as a resuscitation end point. These needs are
real in infants and very young children whose renal concentrating abilities are not completely mature.
However, in toddlers and older children whose concentrating abilities are more mature, targeting urine
flow of 0.5–1 cc/kg/hour results in overall fluid requirements closer to that of an adult and less overall
edema. Patients with inhalation injury have demonstrated to have overall volume requirements greater
than that predicted by standard formulae, possibly secondary to release of vasoactive mediators from
injured burned parenchyma. During the first 24 hours, Ringer’s lactate is the primary resuscitative fluid.
Because children less than 10 kg can develop hypoglycemia if glucose is not administered, Ringer’s
lactate or half normal saline with 5% dextrose at a maintenance rate is given along with the reduced
amount of Ringer’s lactate. Dextrose containing fluids should not be given as a primary resuscitative
fluid in adults, as hyperglycemia and osmotic diuresis will result.

78. Which of the following statement(s) is/are true concerning techniques of burn excision, and
temporary and definitive wound closure?

a. Techniques to conserve blood include subeschar injection of dilute epinephrine solution,


exsanguination of the extremity and inflation of a pneumatic tourniquet

b. Fresh or cryopreserved human allograft is usually rejected within 2 to 4 weeks


c. A common use for human allograft is as a physiologic cover for selected clean superficial wounds as
they epithelialize

d. A donor site can only serve as a single source for autograft

Answer: a, b

A common argument against the policy of early burn wound excision is the prodigious blood loss which
has been associated with these procedures. However, modern blood conserving practices as well as
earlier excision of wounds have diminished this concern. Tangential excision of the torso, neck and head
are done after subeschar injection of dilute epinephrine solutions. Tangential excisions of the
extremities are done after exsanguination and inflation of a pneumatic tourniquet. Once necrotic eschar
is excised to a bed of viable tissue, immediate biologic closure is mandatory. Ideally, immediate
autografting is performed. When donor sites are insufficient for this purpose, a temporary biologic cover
must be chosen while awaiting healing of donor sites for further use. Such covers should prevent
desiccation and provide a vapor and bacterial barrier over the excised wound. Fresh or cryopreserved
human allograft is most appropriate for this use. Once placed on a viable wound bed, it will vascularize
and provide physiologic wound closure until rejected 2 to 4 weeks later at which time or before, it is
replaced with reharvested autograft. A second common use for biologic dressings is a physiologic cover
for selected clean superficial wounds as they epithelialize, which minimizes the pain associated with
open partial thickness burns. Allograft, screened for malignant and infectious diseases, a precarious
resource, is however, not commonly used as a biologic dressing in these circumstances. For this
purpose, reconstituted porcine xenograft should be used.

79. Which of the following statement(s) is/are true concerning topical antimicrobials in common use in
the United States today?

a. Of the common topical antimicrobials, only mafenide acetate is painful upon application

b. The use of 0.5% silver nitrate is associated with trans-eschar leeching of sodium and potassium from
the wound

c. Silver sulfadiazine has the best eschar penetration

d. Silver sulfadiazine, mafenide acetate, and 0.5% silver nitrate all have a broad spectrum activity,
however, only silver nitrate has anti-fungal activity

Answer: a, b, d
THREE COMMON TOPICAL MEDICATIONS USED IN THE UNITED STATES

Agent Characteristics

Silver sulfadiazine Painless on application

Fair to poor eschar penetration

No metabolic side effects

Broad antibacterial spectrum

Mafenide acetate Painful on application

Excellent eschar penetration

Carbonic anhydrase inhibitor

Broad-spectrum antibacterial

0.5% Silver nitrate Painless on application

Poor eschar penetration

Leeches electrolytes

Broad-spectrum antibacterial and antifungal

80. The anthropometric changes observed as a patient progresses from infancy to adulthood include
which of the following statement(s)?

a. The major anthropometric changes involve the head and torso

b. A decrease in the relative size of the head from 18% to 9% of the body’s surface area occurs

c. The total surface area of the legs increases from 14 to 18%

d. The upper extremities increase to 12% of the body surface area

Answer: b, c
An accurate assessment of burn size can be made early and is important to the initial management as
resuscitative fluid administration is primarily determined by overall burn size. Burn size in children is
best estimated with an age-specific chart, because the child’s body proportions change with growth. The
major anthropometric change involves the head and legs. The infant’s head represents 18% of the total
body surface and legs 14%. In older adolescents and adults, the head represents 9% of the body surface
and the legs 18%. Each upper extremity in the adult is usually considered to represent 9% of the total
body surface area.

81. Arguments in favor of early wound excision include which of the following statement(s)?

a. Enhanced survival is seen in patients with large burn injuries

b. Hospital stays can be shortened with this technique

c. Early burn excision results in fewer painful dressing changes

d. A decrease in duration and intensity of the hypermetabolic response is observed

Answer: a, b, c, d

Early removal of extensive areas of devitalized tissue with immediate biologic closure of the resulting
wounds is the core surgical objective of the first burn week. The policy of early excision is now widely
practiced in the United States and is carried out as an excision of the entire wound coincident with fluid
resuscitation, or more commonly, by staged excision of all deep partial and full thickness components of
the wound (less the face, palms, soles, and genitals) over the first three to seven days after injury. The
increasing popularity of early excision is based upon several documented and perceived advantages over
the traditional approach of allowing eschar to be liquefied by bacterial proteases until separation occurs,
leaving a bed of granulation tissue which is subsequently autografted. Documented advantages include
improved survival in patients with injuries involving more than 30–40% of the body surface, truncated
hospital stays, lowered cost, and fewer painful dressing changes. Although not proven, conventional
wisdom suggests that a decrease in the duration and intensity of the hypermetabolic response,
improved immunologic function and less hypertrophic scarring result from early excision.

82. Which of the following are accepted adjuncts in the management of hypertrophic scar?
a. Local steroid injection

b. Compression garments

c. Topical silicone

d. Release or excision with autografting

e. Topical platelet-derived growth factor

Answer: a, b, c, d

Hypertrophic scar formation is a major source of long-term morbidity after burns. All healed and grafted
burns become hypervascular shortly after successful epithelialization. Wounds destined to become
hypertrophic develop a second surge of neovascularization between 9 and 13 weeks. Wounds that are
most commonly associated with hypertrophy are deep dermal burns that heal in three or more weeks
and full thickness wounds that heal by contraction and epithelial spread from wound edges. Current
tools used in the prevention of hypertrophic scars include compression garments, topical silicone sheets,
steroid injections, and release or excision and autografting.

83. Which of the following statement(s) is/are true concerning carbon monoxide and cyanide exposure?

a. A normal oxygen saturation by standard transmission pulse oximetry precludes the possibility of
significant carboxyhemoglobinemia

b. Most patients with cyanide exposure require administration of sodium thiosulfate

c. The half-life of carbon monoxide is reduced by a factor of 5 with ventilation with 100% oxygen

d. Even if fire victims are well ventilated with high concentrations of oxygen by emergency response
personnel from the time of extrication, carboxyhemoglobin values are frequently greater than 10% on
initial evaluation

Answer: c
Both carbon monoxide and cyanide are commonly inhaled by victims of closed space fires. Patients with
significant amounts of carboxyhemoglobin suffer from a marked reduction in their ability to deliver
oxygen to peripheral tissues despite a normal arterial partial pressure of oxygen. Its 2.5 hour half-life is
reduced by a factor of 5 by ventilation with 100% oxygen. Fire victims who are well ventilated with high
concentrations of oxygen by emergency response personnel from the time of extrication commonly
have normal carboxyhemoglobin values (< 5%) on initial evaluation despite significant exposure to
carbon monoxide at the time of injury. Carboxyhemoglobin is not sensed by standard transmission pulse
oximetry, so a normal oxygen saturation on such a monitor does not preclude the possibility of
significant carboxy-hemoglobinemia.

Hydrogen cyanide, which is commonly present in the smoke of structural fires, interferes with oxidative
metabolism at the cellular level resulting in lactic acidosis. With proper ventilation and fluid
resuscitation, the cyanide-induced acidosis corrects in most cases and specific treatment with sodium
thiosulfate is not generally required.

84. Valid points concerning the initial physical examination in a burn patient include which of the
following statement(s)?

a. Patients should be examined in a warm environment to prevent hypothermia

b. All corneal injuries are obvious on initial physical examination

c. Inhalation injury is suggested by the presence of singed facial hair and nasal vibrissae, carbonaceous
sputum, and a hoarse voice

d. Blistering in or around the mouth may suggest hot liquid aspiration in small children

Answer: a, c, d

An organized approach to serious burn injuries facilitates achieving the optimal outcome and begins
with a systematic initial evaluation that includes a primary survey, effective vascular and airway access,
and a systematic secondary survey. Many burn patients have sustained concurrent injuries and the
initial evaluation should therefore be approached as any multiple trauma patient. After evaluating and
securing the airway, while maintaining control of the cervical spine, breathing mechanics are assessed, a
rough estimate is made of circulating volume, the level of consciousness is documented, and the patient
is completely exposed. This should be done in a warm environment to avoid hypothermia. The burn
specific secondary survey includes a complete history, vital signs, a detailed physical examination and
laboratory and radiologic studies appropriate for the mechanism of injury. The patient’s neurologic
status should be carefully documented early during the evaluation, because many patients will become
progressively obtunded secondary to the administration of analgesics and sedatives and from
intravascular volume depletion. The corneal epithelium and globes should be examined prior to the
development of adnexal edema which will render adequate examination more difficult. Major corneal
epithelial burns are obvious by the opaque appearance that results. More subtle defects are apparent
only after staining with topical fluorescein. Upper airway injuries are suspected by the presence of a
hoarse voice, burns to the lips or tongue, singed facial hair and nasal vibrissae, or carbonaceous sputum.
Hot liquid aspiration may complicate facial scald burns with small children, and should be suspected if
there is blistering in or around the mouth.

85. The systemic response to a significant burn includes accelerated fluid losses. Which of the following
statement(s) is/are true concerning tissue edema following a burn?

a. Edema in tissue immediately surrounding the burn is secondary to local disruption of the capillary bed

b. Edema in tissue immediately surrounding the burn is secondary to the local release of vasoactive
mediators such as prostaglandins, thromboxane A2, and reactive oxygen radicals

c. Tissue edema following a burn occurs only in the tissues at or immediately adjacent to the burn

d. Pulmonary changes following a burn occur only secondary to excessive fluid administration

Answer: b

The systemic response to burning is driven by the loss of the skin’s barrier functions with accelerated
fluid losses and decreased host resistance to infection, release of mediators from the injured tissue with
secondary interstitial edema and organ dysfunction and from bacterial overgrowth within the eschar
with the resulting systemic sequelae. Edema in tissue immediately surrounding the burn occurs
secondary to local release of vasoactive mediators, such as prostaglandins, thromboxane A2 and
reactive oxygen radicals. When burn size exceeds 20% to 30% of the body surface, clinically significant
interstitial edema is seen in distant soft tissue secondary to a combination of wound released mediators
and hypoproteinemia. These distant microvascular effects also have the ability to interfere with the
function of organ systems not directly injured by the burning process, explaining the frequent
occurrence of pulmonary and other organ dysfunction in patients with large burns.
86. A patient sustains a high voltage electrical injury to the upper extremity. Which of the following
statement(s) is/are true concerning peripheral perfusion to the injured arm?

a. Evidence of peripheral ischemia would be evident within the first few hours after injury

b. Physical signs of diminished blood flow include a progressive increase in the extremity’s consistency
and a decrease in distal temperature

c. A bedside escharotomy is an appropriate treatment

d. An immediate fasciotomy performed in the operating room may be necessary

Answer: b, d

Regular assessment and documentation of peripheral perfusion is crucial during the first post-injury
days. Blood flow can be compromised by constricting circumferential eschar as subeschar tissues
become progressively edematous or by progressive intracompartmental edema in patients with
electrical or deep thermal burns. Both are detected by the development of a progressive increase in the
extremity’s consistency and a decrease in its distal temperature. Pulsatile doppler signals in the lower
pressure distal vasculature, such as the plantar arch and digital vessels, should be documented hourly.
The loss of pulsatile doppler signals is consistent with an increase in tissue pressure if intravascular
volume is adequate. Although lesser voltages may cause local destructive injuries without systemic
sequelae, patients exposed to higher voltage (> 1000 volts), present with a combination of deep tissue
injuries secondary to the passage of current, locally destructive entrance and exit wounds, and other
local and systemic effects. Compartmental pressure elevation, secondary to edema of injured muscle,
can result in additional ischemic injury if compartments are not promptly released by fasciotomy.

You might also like